SlideShare a Scribd company logo
Trang 1
BỘ GIÁO DỤC VÀ ĐÀO TẠO
VIỆN NGHIÊN CỨU CAO CẤP VỀ TOÁN
BÀI BÁO CHỦ ĐỀ HÌNH HỌC PHẲNG
ĐỀ TÀI
ỨNG DỤNG CỦA PHƯƠNG TÍCH, TRỤC ĐẲNG PHƯƠNG
VÀO BÀI TOÁN HÌNH HỌC PHẲNG
Văn Phú Quốc
GV. Trường THPT chuyên Nguyễn Bỉnh Khiêm,
tỉnh Quảng Nam
Phương tích và trục đẳng phương là một chủ đề quen thuộc và hấp dẫn trong
hình học phẳng. Các kết quả của nó vô cùng đơn giản, tự nhiên nhưng lại ảnh hưởng
sâu sắc đến các nội dung quan trọng như: tính độ dài, góc, chứng minh đẳng thức-bất
đẳng thức hình học, tập hợp điểm, điểm-đường cố định, quan hệ vuông góc, quan hệ
đồng quy-thẳng hàng…Tìm được mối liên hệ giữa phương tích và trục đẳng phương
với các nội dung trên sẽ giúp người làm toán hướng đến những lời giải hay, đẹp, gọn
gàng và ấn tượng.
A. CƠ SỞ LÝ THUYẾT
I. Phương tích của một điểm đối với đường tròn (Power of a point)
1. Định lý 1. Cho đường tròn  
;
O R và điểm M cố định, đặt OM d
 . Một đường
thẳng thay đổi cắt đường tròn tại hai điểm A và B. Khi đó:
2 2 2 2
.
MA MB MO R d R
    .
Chứng minh
Gọi C là điểm đối xứng của A qua tâm O
Vì  0
90
ABC  (góc nội tiếp chắn nửa đường tròn)
nên B là hình chiếu của C trên AM .
Bây giờ ta có
  
. . .
MA MB MA MB MC MA MO OC MO OA
    
  
  
  
 

  
2 2
MO OA MO OA MO OA
    

 
 
 
 
 

2 2 2 2
MO OA d R
   
2. Định nghĩa. Đại lượng không đổi 2 2
.
MA MB d R
  được gọi là phương tích của
điểm M đối với đường tròn  
O , ký hiệu  
/
M O
P . Ta có công thức:
 
2 2
/
.
M O
MA MB d R
  
P
3. Các tính chất
3.1. Tính chất 1
 Nếu ,
A B cố định và .
AB AM const
 thì M cố định.
 Điểm M nằm ngoài    
/
; 0
M O
O R  
P .
B
O
M
A
C
Trang 2
 Điểm M nằm trong    
/
; 0
M O
O R  
P .
 Điểm M nằm trên biên    
/
; 0
M O
O R  
P .
3.2. Tính chất 2
Cho đường tròn  
;
O R và một điểm M nằm ngoài đường tròn  
O . Qua M kẻ cát
tuyến MAB và tiếp tuyến MT với  
O . Khi đó 2 2 2
.
MA MB MT OM R
  
 
.
3.2. Tính chất 3
Cho ,
AB CD là hai đường thẳng phân biệt sao cho AB CD M
  ( , , ,
A B
M C D

 ).
Khi đó nếu . .
MA MB MC MD

  
 

thì bốn điểm , , ,
A B C D cùng nằm trên một đường tròn.
Chứng minh
Giả sử đường tròn ngoại tiếp ABC
 cắt CD tại D . Khi đó:
. .
MA MB MC MD MD MD D D
  
    
  
  
 
.
Vậy bốn điểm , , ,
A B C D cùng nằm trên một đường tròn.
3.3. Tính chất 4
Cho ,
AB MT là hai đường thẳng phân biệt sao cho AB MT M
  (với , ,
M A B T

 ).
Khí đó nếu 2
.
MA MB MT

 
thì đường tròn ngoại tiếp ABT
 tiếp xúc với MT tại T .
4. Một số ví dụ cơ bản
3.1. Ví dụ 1. Cho ABC
 nội tiếp đường tròn  
;
O R và G là trọng tâm ABC
 .
Chứng minh rằng:    
2 2 2
/
1
9
G O
AB BC CA
   
P .
Lời giải
Vì G là trọng tâm ABC
 nên 3OG OA OB OC
  
 
 
 

.
Suy ra  
2 2 2 2
9 2 . . .
OG OA OB OC OAOB OB OC OC OA
     

 
 
 
  

 
2
3 2 . . .
R OAOB OB OC OC OA
   

 
 
   

(1)
Ta có
 
2
2 2 2 2 2 2 2
2 . 2
OAOB OA OB OA OB OA OB AB R AB
        

 
 
 
 
 

.
Tương tự
2 2
2 . 2
OB OC R BC
 

 

; 2 2
2 . 2
OC OA R CA
 
 

Suy ra    
2 2 2 2
2 . . . 6
OAOB OB OC OC OA R AB BC CA
     

 
 
   

(2)
Thay (2) vào (1) ta được:
   
2 2 2 2 2 2 2 2 2 2
1
9 9
9
OG R AB BC CA OG R AB BC CA
          .
Do đó    
2 2 2
/
1
9
G O
AB BC CA
   
P .
[Phương tích này được gọi là phương tích trọng tâm]
Trang 3
3.2. Ví dụ 2. Cho ABC
 nội tiếp đường tròn  
;
O R và H là trực tâm ABC
 .
Chứng minh rằng:  
2
/
8 cos cos cos
H O
R A B C
 
P .
Lời giải
Đối với bài toán này, ta quan tâm đến trường hợp ABC
 nhọn, còn những trường hợp
vuông hoặc tù ta chứng minh tương tự.
Gọi ,
I A lần lượt là giao điểm của AH với  
,
BC O .
Áp dụng định lý sin trong HAB
 ta có
     
0 0
sin 90 sin 180
sin sin
AH AB AH AB
A C
ABH AHB
  
 
.cos 2 cos
cos sin sin
AH AB AB
HA A R A
A C C
     .
Tương tự ta cũng có 2 cos
HB R B
 ; 2 cos
HC R C
 .
Vì   
BHA C BA A
 
  nên BHA
 cân tại B.
Suy ra I là trung điểm A H
 .
Khi đó 
2 2. .cos 4 cos cos
 
  
HA IH HB BHA R B C .
Vì ABC
 nhọn nên  
2
/
. . 8 cos cos cos
H O
HA HA HA HA R A B C
 
    
P .
[Phương tích này được gọi là phương tích trực tâm]
3.3. Ví dụ 3. Cho đường tròn  
;
O R và ba điểm thẳng hàng , ,
A B C . Chứng minh
đẳng thức:      
/ / /
. . . . . 0
A O B O C O
BC CA AB BC CA AB
   
P P P .
Lời giải
Ta có:      
/ / /
. . . . . 0
A O B O C O
BC CA AB BC CA AB
   
P P P
     
2 2 2 2 2 2
. . . . .
OA R BC OB R CA OC R AB BC CA AB
      
 
2 2 2 2
. . . . .
OA BC OB CA OC AB BC CA AB R BC CA AB
      
2 2 2
. . . . .
OA BC OB CA OC AB BC CA AB
   
Ta sẽ chứng minh hệ thức Stewart 2 2 2
. . . . . 0
OA BC OB CA OC AB BC CA AB
   
Trường hợp 1. Điểm O nằm trên đường thẳng chứa ba điểm , ,
A B C .
2 2 2
. . . . .
  
OA BC OB CA OC AB BC CA AB
         
2 2 2
OA OC OB OB OA OC OC OB OA OC OB OA OC OB OA
         
2 2 2 2 2 2 2 2
. . . . . . . .
OA OC OA OB OB OA OB OC OC OB OC OA OA OC OA OB
       
2 2 2 2
. . . . . . . . 0
OB OA OB OC OC OB OC OA OAOB OC OAOB OC
       .
I
C'
B'
A'
H O
B C
A
Trang 4
Trường hợp 2. Điểm O không nằm trên đường thẳng chứa ba điểm , ,
A B C .
Gọi H là hình chiếu của điểm O lên đường thẳng chứa ba điểm , ,
A B C .
2 2 2
. . . . .
  
OA BC OB CA OC AB BC CA AB
     
2 2 2 2 2 2
. . . .
OH HA BC OH HB CA OH HC AB BC CA AB
      
   
2 2 2 2
. . . . .
OH BC CA AB AH BC BH CA CH AB BC CA AB
      
0
 .
Vậy      
/ / /
. . . . . 0
A O B O C O
BC CA AB BC CA AB
   
P P P .
4. Phương tích trong hệ tọa độ Descartes
Trong mặt phẳng với hệ tọa độ Oxy , cho điểm  
0 0
;
M x y và đường tròn
  2 2
: 2 2 0
C x y ax by c
     .
Đặt   2 2
, 2 2
f x y x y ax by c
     . Khi đó
    2 2
0 0 0 0 0 0
/
; 2 2
M C
f x y x y ax by c
     
P .
II. Trục đẳng phương của hai đường tròn (Radical axis) và tâm đẳng phương
(radical center)
1. Định lý 2. Cho hai đường tròn không đồng tâm    
1 1 2 2
; , ;
O R O R . Tập hợp điểm M
có phương tích đối với hai đường tròn bằng nhau là một đường thẳng. Đường thẳng
này được gọi là trục đẳng phương của hai đường tròn  
1
O và  
2
O .
Chứng minh
Giả sử điểm M có cùng phương tích đối
với hai đường tròn  
1
O và  
2
O .
Gọi H là hình chiếu của điểm M lên
đường thẳng 1 2
O O và I là trung điểm đoạn
thẳng 1 2
O O .
Khi đó
   
2 2 2 2 2 2
1 2 1 2
MH HO MH HO R R
    
2 2 2 2
1 2 1 2
HO HO R R
   
H
O
A C
B
I
O1
O2
H
M
Trang 5
   2 2
1 2 1 2 1 2
HO HO HO HO R R
    
2 2
2 2 1 2
2 1 1 2
1 2
.2
2
R R
O O HI R R IH
O O

     .
Vì H là điểm cố định nên tập hợp các điểm M có cùng phương tích với hai đường
tròn là đường thẳng qua H và vuông góc với 1 2
O O .
2. Các tính chất về trục đẳng phương
Xét hai đường tròn  
1
O và  
2
O . Từ kết quả ở định lý 2 ta suy ra một số tính chất
sau:
2.1. Tính chất 1. Trục đẳng phương của hai đường tròn vuông góc với đường thẳng
nối tâm.
2.2. Tính chất 2. Nếu hai đường tròn cắt nhau tại A và B thì AB chính là trục đẳng
phương của chúng.
2.3. Tính chất 3. Nếu    
1 2
/ /
M O M O

P P thì đường thẳng qua M vuông góc với 1 2
O O là
trục đẳng phương của hai đường tròn.
2.4. Tính chất 4. Nếu    
1 2
/ /
M O M O

P P và    
1 2
/ /
N O N O

P P thì đường thẳng MN chính
là trục đẳng phương của hai đường tròn.
2.5. Tính chất 5. Nếu    
1 2
/ /
M O M O

P P ;    
1 2
/ /
N O N O

P P và    
1 2
/ /
P O P O

P P thì ba điểm
, ,
M N P thẳng hàng.
2.6. Tính chất 6. Nếu  
1
O và  
2
O tiếp xúc nhau tại A thì đường thẳng qua A và
vuông góc với 1 2
O O chính là trục đẳng phương của hai đường tròn.
3. Cách dựng trục đẳng phương của hai đường tròn không đồng tâm
Trong mặt phẳng, cho hai đường tròn không đồng tâm  
1
O và  
2
O . Xét các khả
năng có thể xảy ra:
3.1. Khả năng 1.      
1 2 ,
O O A B
  . Khi đó đường thẳng AB chính là trục đẳng
phương của hai đường tròn.
3.2. Khả năng 2.  
1
O và  
2
O tiếp xúc nhau tại A. Khi đó tiếp tuyến chung tại A
chính là trục đẳng phương của hai đường tròn.
3.3. Khả năng 3.    
1 2
O O
   . Ta dựng theo các bước dưới đây:
 Bước 1: Dựng đường tròn  
3
O sao cho      
3 1 ,
O O A B
  và
     
3 2 ,
O O C D
  .
 Bước 2: AB CD M
  .
 Bước 3: Dựng đường thẳng qua M và vuông góc với đường thẳng 1 2
O O chính
là trục đẳng phương của  
1
O và  
2
O .
4. Trục đẳng phương trong hệ tọa độ Descartes
Trong mặt phẳng với hệ tọa độ Oxy , cho hai đường tròn không đồng tâm:
  2 2
1 1 1 1
: 2 2 0
C x y a x b y c
     và   2 2
2 2 2 2
: 2 2 0
C x y a x b y c
    
Trang 6
(với 2 2
0
i i i
a b c
   , 1,2
i  ).
Giả sử điểm  
0 0
;
M x y .
Ta có    
1 2
2 2 2 2
0 0 2 0 2 0 2 0 0 2 0 2 0 2
/ /
2 2 2 2
M C M C
x y a x b y c x y a x b y c
          
P P
   
1 2 0 1 2 0 1 2
2 2 0
a a x b b y c c
       .
Vậy trục đẳng phương của hai đường tròn  
1
C và  
2
C có phương trình:
   
1 2 1 2 1 2
2 2 0
a a x b b y c c
      .
5. Định lý 3. Cho ba đường tròn    
1 2
,
O O và  
3
O . Khi đó ba trục đẳng phương của
các cặp đường tròn hoặc trùng nhau hoặc song song hoặc cùng đi qua một điểm. Điểm
đó là tâm đẳng phương của ba đường tròn.
Chứng minh
Gọi ij
 là trục đẳng phương của hai đường tròn  
i
O và  
j
O . Ta xem xét các trường
hợp sau đây:
Trường hợp 1. Giả sử có một cặp đường thẳng song song. Không giảm tính tổng quát,
ta có thể giả sử 12
 // 23
 . Ta có 12 1 2 23 2 3
,
O O O O
    . Điều này dẫn đến 1 2 3
, ,
O O O
thẳng hàng. Lại có 13 1 3
O O
  nên 13
 // 23
 // 12
 .
Trường hợp 2. Giả sử có một cặp đường thẳng cắt nhau. Không giảm tính tổng quát, ta
có thể giả sử 12 23 M
    .
13
23
12
M
O1
O2
O3
Bây giờ ta có
   
   
   
1 2
1 3
2 3
1
/ /
/ 3
/
/ /
M O M O
M O M O
M O M O
M




  




P P
P P
P P
.
Bắt đầu từ đây, ta suy ra:
- Nếu có hai đường thẳng trùng nhau thì đó cũng là trục đẳng phương của cặp đường
tròn còn lại.
- Nếu hai trục đẳng phương chỉ cắt nhau tại một điểm thì điểm đó cũng thuộc trục
đẳng phương còn lại.
Trang 7
6. Các tính chất về tâm đẳng phương
6.1. Tính chất 1. Nếu ba đường tròn đôi một cắt nhau thì các dây cung cùng đi qua
một điểm.
6.2. Tính chất 2. Nếu ba trục đẳng phương song song hoặc trùng nhau thì tâm của ba
đường tròn thẳng hàng.
6.3. Tính chất 3. Nếu ba đường tròn cùng đi qua một điểm và có các tâm thẳng hàng
thì các trục đẳng phương trùng nhau.
7. Một số ví dụ cơ bản
7.1. Ví dụ 1. Cho ABC
 , một đường thẳng song song với BC cắt ,
AB AC tương ứng
tại ,
D E . Xác định trục đẳng phương của các đường tròn đường kính ,
BE CD.
Lời giải
Gọi    
1 2
,
C C lần lượt là các đường tròn đường
kính ,
BE CD.
Gọi , ,
AA BB CC
   là các đường cao của ABC

và H là trực tâm.
Dễ thấy các tứ giác HA CB
 , HA BC
  nội tiếp,
cho nên
. .
. .
. .
AB AC AH AA
AB AC AC AB
AC AB AH AA
  


 
 

 



(1)
Vì DE // BC nên
AC AB
AE AD
 (2)
Từ (1) và (2) suy ra . '.
AB AE AC AD
 
   
1 2
/ /
A C A C
 
P P
Suy ra A nằm trên trục đẳng phương của hai đường tròn  
1
C và  
2
C .
Hơn nữa, ta thấy AA vuông góc với đường nối tâm của hai đường tròn  
1
C và  
2
C
Vậy AA là trục đẳng phương của  
1
C và  
2
C .
7.2. Ví dụ 2. Cho ABC
 có đường cao BD và CE cắt nhau tại H . Gọi M là trung
điểm BC , N là giao điểm của DE và BC . Chứng minh rằng: NH AM
 .
Lời giải
Gọi ,
O I lần lượt là trung điểm của các
đoạn thẳng ,
AH MH .
Ta có:
   
DEH DAH DBC FEH
  
   
2. 2.
FED FEH DBC DMC
   
Suy ra tứ giác EDMF nội tiếp.
Điều đó dẫn đến . .
NE ND NF NM
 tức
điểm N nằm trên trục đẳng phương của
đường tròn  
;
O OH và đường tròn  
;
I IH .
E
D
A'
B'
C'
H
B
A
C
I
O
N M
D
E
F
H
B
A
C
Trang 8
Mặt khác,    
; ;
H O OH I IH
  . Do đó NH chính là trục đẳng phương của hai
đường tròn  
O và  
I .
Suy ra NH OI
 , mà OI // AM nên NH AM
 .
Vậy ta hoàn tất việc chứng minh.
7.3. Ví dụ 3. Cho đường tròn  
O đường kính AB và CD . Tiếp tuyến tại B của  
O
cắt AC tại E , DE cắt  
O tại điểm thứ hai F . Chứng minh rằng , ,
AF BC OE đồng
quy.
Lời giải
Ký hiệu    
1 2
,
C C tương ứng là đường tròn ngoại
tiếp của các tam giác ,
AEF BCE .
Ta có ,
AF BC tương ứng là trục đẳng phương của
 
O và  
1
C ,  
O và  
2
C .
Mặt khác   
OAF FDB FEA
  ;  
OBC CEB
 .
Suy ra ,
OA OB lần lượt là tiếp tuyến của    
1 2
,
C C .
Kết hợp với 2 2
OA OB
 ta suy ra OE là trục đẳng
phương của  
1
C và  
2
C .
Suy ra , ,
AF BC OE đồng quy tại tâm đẳng phương
của ba đường tròn. [tư tưởng của định lý 3].
B. MỘT SỐ ỨNG DỤNG
I. Ứng dụng 1: Chứng minh một số định lý về hình học phẳng
1.1. [Định lý Brianchon]. Cho ABCDEF là một lục giác ngoại tiếp đường tròn  
O .
Khi đó , ,
AD BE CF đồng quy.
Chứng minh
Gọi , , , , ,
G H I J K L tương ứng là tiếp điểm của
, , , , ,
AB BC CD DE EF FA với  
O .
Trên các tia , , , , ,
KF HB GB JD ID LF lần lượt
lấy các điểm , , , , ,
P S Q R N M sao cho
KP SH GQ JR IN LM
     .
Dựng đường tròn  
1
O tiếp xúc với ,
EF CB tại
,
P S ; đường tròn  
2
O tiếp xúc với ,
AF CD tại
,
M N và đường tròn  
3
O tiếp xúc với ,
AB ED
tại ,
Q R.
Ta có FP PK FK LM LF FM
    
CS SH HC IN IC CN
    
F
E
O
B
A
D C
N
M
R
Q
S
P
C
D
E
F
A B
I
J
G
H
K
L
O1
O3
O2
Trang 9
Suy ra FC là trục đẳng phương của các đường tròn  
1
O và  
2
O .
Chứng minh tương tự, ta cũng thu được: AD là trục đẳng phương của các đường tròn
 
2
O và  
3
O ; BE là trục đẳng phương của  
3
O và  
1
O .
Áp dụng định lý 3 [về tâm đẳng phương] ta đi đến , ,
AD BE CF đồng quy.
1.2. [Hệ thức Euler]. Cho ABC
 nội tiếp đường tròn  
;
O R và ngoại tiếp đường tròn
 
;
I R . Chứng minh rằng 2 2
2
OI R Rr
  .
Chứng minh
Gọi E là tiếp điểm của AB và  
;
I r ;
gọi  
' ;
A AI O R
  .
Ta có

sin sin
2
IE r
IA
A
IAE
  (do AA là phân giác

BAC ).
Mặt khác,   
  
0
180
2 2
A B
BIA AIE BIE IBA
 
     
A IB

 cân tại A IA BA
  
  .
Áp dụng sin cho BAA
 ta có:
2 sin 2 sin
2 2
A A
BA R IA R
 
   .
Vì I là điểm nằm bên trong  
;
O R nên
 
2 2 2 2
/
. .2 sin 2 2 2
2
sin
2
I O
r A
IA IA R Rr OI R Rr OI R Rr
A

            
P .
1.3. [Định lý Fuss]. Cho tứ giác ABCD nội tiếp đường tròn  
;
O R và ngoại tiếp
đường tròn  
;
I r . Đặt 0
OI d
  . Khi đó
   
2 2 2
1 1 1
r
R d R d
 
 
.
Chứng minh
Kéo dài ,
BI DI cắt  
O tại ,
M N .
Ta có:
 
 
MNC IBC
NMC IDC
 





     
  0
1
90
2
MNC NMC IBC IDC ABC ADC
      
O
 là trung điểm đoạn thẳng MN .
Sử dụng công thức đường trung tuyến trong IMN
 có
E
A'
I
O
C
A
B
I
F
E
O
C
A
D
B
M
N
Trang 10
2 2 2 2 2
2 2 2
2 4 2
IM IN MN IM IN
d OI R
 
    
Suy ra
2 2
2 2
2
IM IN
R d

  .
Do đó
   
 
   
 
2 2 2 2
2 2 2 2
2 2
/
2
1 1
I O
R d IM IN
R d R d R d
 
  
   P
2 2
2 2 2 2 2 2
1 1
. .
IM IN
IM IB IN ID IB ID
   
 
 
2 2
2 2
2 2 2
sin sin
sin sin 1
2 2
2
ABC ADC
IBE IDF
IE IF r

    .
II. Ứng dụng 2: Chứng minh quan hệ vuông góc
2.1. [Mathematical Olympiad Summer Program (MOSP) 1995]. Cho ABC
 có
đường cao BD và CE cắt nhau tại H . M là trung điểm của BC , N là giao điểm của
DE và BC .
Chứng minh rằng NH AM
 .
Lời giải
Gọi ,
O I tương ứng là trung điểm của
,
AH MH .
Ta có    
DEH DAH DBC FEH
  
   
2. 2.
FED FEH DBC DMC
   
tức giác EDMF nội tiếp
. .
NE ND NF NM N
   nằm trên trục
đẳng phương của đường tròn đường kính
MH và đường tròn đường kính AH .
Mặt khác,    
H O I NH
   là trục
đẳng phương của  
O và  
I .
Suy ra NH OI
 . Mà OI // AM (do OI là đường trung bình AHM
 ) nên
NH AM
 .
2.2. [Iran 1996]. Cho hai điểm ,
D E tương ứng nằm nằm trên các cạnh ,
AB AC của
ABC
 sao cho DE // BC . Gọi P là điểm bất kì nằm bên trong ABC
 ; các đường
thẳng ,
PB PC lần lượt cắt DE tại F và G . Gọi 1 2
,
O O lần lượt là tâm đường tròn
ngoại tiếp các tam giác ,
PDG PFE . Chứng minh 1 2
AP O O
 .
O
I
N M
E
D
F
H
A
B
C
Trang 11
Lời giải
Gọi M , N lần lượt là giao điểm thứ
hai của AB với  
1
O , của AC với
 
2
O .
Ta có   
PMD PGD PCB
   tứ giác
BPCM nội tiếp.
Tương tự ta cũng chứng minh được
tứ giác BPCN nội tiếp.
Điều đó dẫn đến tứ giác BMNC nội
tiếp. Mà DE // BC cho nên ta thu
được tứ giác MDEN nội tiếp.
Áp dụng định lý 3 [về tâm đẳng
phương] cho các đường tròn
   
,
DGP PEF và  
DENM ta có
DM EN A
  nằm trên trục đẳng
phương của  
1
O và  
2
O .
P nằm trên trục đẳng phương AP
 là trục đẳng phương của  
1
O và  
2
O .
Do đó 1 2
AP OO
 (điều phải chứng minh).
2.3. [IMO 1985]. Cho ABC
 . Đường tròn tâm O đi qua các điểm ,
A C và lại cắt các
đoạn ,
AB BC theo thứ tự tại hai điểm phân biệt ,
K N . Giả sử đường tròn ngoại tiếp
của các tam giác ABC và KBN cắt nhau tại B và M . Chứng minh  0
90
OMB  .
Lời giải
Gọi D AC KN
  .
Ta có
   
  
 
0 0
180 180
KMA BMA KMB ACB KNB
     
  
KNB ACB KDA
   .
tứ giác AKMD nội tiếp
   
0 0
180 180
AMD AKD ACB AMB
     
Do đó , ,
B M D thẳng hàng.
Gọi R là bán kính của đường tròn  
O .
Ta có  
2 2
2 2
. .
*
. .
BM BD BK BA BO R
DM DB DC DA DO R
   


  


  2 2 2
. 2
BM DM BD BO DO R
    
2 2 2 2
2
BD BO DO R
    .
M
N
O2
O1
P
E
A
B C
D
F
G
M
D
K
N
A
B
C
O
Trang 12
Từ (*) ta suy ra:
2 2
2 2 2 2
2 2 BO R DO R
BM DM
BD BD
   
 
  
   
   
  
2 2 2 2 2
2
2
BO DO R BO DO
BD
  


2 2 0
90
BO DO BMO
    .
2.4. [Belarus 2000]. Gọi P là giao điểm của đường chéo AC và BD của tứ giác lồi
ABCD trong đó AB AC BD
  . Gọi O và I lần lượt là tâm đường tròn ngoại tiếp và
nội tiếp của ABP
 . Nếu minh rằng nếu O không trùng với I thì OI CD
 .
Lời giải
Đầu tiên ta chứng minh một luận đề hết sức hữu
ích về hai đường vuông góc. Cho hai đường XY
và UV , đặt ,
X Y
  lần lượt là chân đường vuông
góc của X và Y tương ứng, nối đường thẳng
UV . Sử dụng trực tiếp khoảng cách, XY UV

nếu và chỉ nếu UX X V UY Y V
   
   .
Vì UX X V UV UY Y V
   
    , phép tính trên
đạt được nếu và chỉ nếu
2 2 2
UX X V UY Y V
   
   hay
2 2 2 2
UX XV UY YV
   .
Do đó nó thỏa mãn đẳng thức
2 2 2 2
DO CO DI CI
   .
Đặt ,
AB AC BD p PC a
    và PD b
 .
Thế thì AP p a
  và BP p b
  . Gọi R là bán
kính đường tròn ngoại tiếp ABP
 . Sử dụng định
lý về phương tích của một điểm, ta có 2 2
.
pb DP DB DO R
   .
Tương tự, 2 2
pa CO R
  .
Do đó  
2 2
DO CO p b a
   .
Vì ABD
 là cân với BA BD
 và I nằm trên đường phân giác của 
ABD ID IA
  .
Ngoài ra IB IC
 . Gọi T là điểm tiếp xúc của đường tròn nội tiếp ABC
 với cạnh
AB . Thế thì
2 2
AB AP BP p b a
AT
   
  và
2
p a b
BT
 
 bởi vì
2 2 2 2
,
IT AB AI BI AT BT
    . Đặt các tham số lại với nhau chúng ta thấy rằng
    
2 2 2 2 2 2 2 2
.
DI CI AI BI AT BT AT BT AT BT p b a DO CO
           
Suy ra OI CD
 .
III. Ứng dụng 3: Chứng minh quan hệ đồng quy, thẳng hàng
3.1. [Vietnam TST 2009]. Cho ABC
 nhọn nội tiếp đường tròn  
O . Các điểm
1 1 1
, ,
A B C lần lượt là chân đường vuông góc của , ,
A B C xuống các cạnh đối diện. Các
điểm 2 2 2
, ,
A B C đối xứng với 1 1 1
, ,
A B C qua trung điểm , ,
BC CA AB . Đường tròn ngoại
T
I
O
P
D
C
B
A
Trang 13
tiếp các tam giác 2 2 2 2 2 2
, ,
AB C BC A CA B cắt  
O lần thứ hai tại 3 3 3
, ,
A B C . Chứng minh
các đường thẳng 1 3 1 3 1 3
, ,
A A B B C C đồng quy.
Lời giải
Gọi  
;
I R là đường tròn ngoại tiếp 2 2
AB C
 ,
M là trung điểm AB và 1 3
G AM A A
  .
Ta có 1 1
1 2 1 2
. .
,
AC AB AB AC
AC BC AB CB
 


 


2 2
. .
BC BA CB CA
 
2 2 2 2
BI R CI R
   
BI CI
 
IO BC
 
Mà 3
IO AA
 cho nên BC // 3
AA .
Dễ dàng có 1 3
1
2
A M AA
 .
Theo định lý Thales ta có 3
1
2
AA
AG
MG A M
  .
Suy ra G là trọng tâm ABC
 .
Chứng minh tương tự, ta cũng suy ra được 1 3 1 3
,
B B C C cũng đi qua G .
Vậy 1 3 1 3 1 3
, ,
A A B B C C đồng quy tại trọng tâm G của ABC
 .
3.2. [IMO Shortlist 2013, Thailand]. Cho ABC
 nhọn với H là trực tâm. Gọi W là
điểm trên cạnh BC . Gọi ,
M N tương ứng là chân đường cao kẻ từ đỉnh B và C . Gọi
1
 là đường tròn ngoại tiếp BWN
 và X là điểm trên 1
 đầu mút của đường kính,
đối diện với W . Tương tự, 2
 là đường tròn ngoại tiếp CWM
 và Y là điểm trên 2

đầu mút của đường kính, đối diện với W . Chứng minh rằng , ,
X Y H thẳng hàng.
Lời giải
Gọi L là chân đường cao kẻ từ A và Z là
giao điểm thứ hai khác W của hai đường tròn
1 2
,
  .
Ta sẽ chứng minh , ,
X Y Z và H cùng nằm
trên một đường thẳng.
Từ   0
90
BNC BMC
  ta suy ra , ,
B C N và
M cùng nằm trên một đường tròn, gọi đường
tròn này là 3
 .
Để ý rằng , ,
WZ BN CM tương ứng là trục
đẳng phương của các cặp đường tròn
1
 và 2
 ; 1
 và 3
 ; 2
 và 3
 .
Z
N
L
M
H
A
B C
W
X
Y
G
A3
I
C2
B2
M
A1
B1
C1
O
C
B
A
A2
Trang 14
Thế thì A BN CM
  là tâm đẳng phương của ba đường tròn và do đó WZ qua A.
Vì ,
WX WY tương ứng là đường kính của 1
 và 2
 nên ta có   0
90
WZX WZY
  , vì
vậy ,
X Y nằm trên đường thẳng qua Z , vuông góc với WZ .
Vì tứ giác BLHN có hai góc đối diện vuông nên nội tiếp đường tròn. Từ phương tích
của điểm A đối với các đường tròn 1
 và  
BLHN ta được:
. . .
AL AH AB AN AW AZ
  .
Nếu H nằm trên đường thẳng AW thì H Z
 lập tức. Mặt khác, vì
AZ AL
AH AW
 nên
các tam giác ,
AHZ AWL đồng dạng.
Vì thế,   0
90
HZA WLA
  . Vậy H cũng nằm trên đường thẳng XYZ .
3.3.[Iran NMO 2001]. Cho ABC
 nội tiếp đường tròn  
O . Gọi    
, a
I I lần lượt là
đường tròn nội tiếp, bàng tiếp góc A. Giả sử a
I I cắt BC và  
O lần lượt tại ,
A M

 
M A

 . Gọi N là trung điểm cung 
MBA và , a
NI NI cắt  
O lần lượt tại ,
S T .
Chứng minh rằng , ,
S T A thẳng hàng.
Lời giải
  
   
  
1 1
2 2
NTS NA AS NM AS NIM
    
 
a a
I TS I IS
   tứ giác a
I TIS nội tiếp đường
tròn và ký hiệu đường tròn này là  
1
 .
Mặt khác,   0
90
a a
IBI ICI
  nên a
IBI C nội
tiếp đường tròn và ký hiệu đường tròn này là
 
2
 .
Nhận thấy , ,
a
I I BC TS tương ứng là trục đẳng
phương của các cặp đường tròn  
1
 và  
2
 ,
 
O và  
2
 ,  
O và  
1
 . Do đó , ,
a
I I BC TS
đồng quy tại A. Vậy ba điểm , ,
S T A thẳng
hàng.
3.4. [IMO 1995]. Trên đường thẳng d lấy bốn điểm , , ,
A B C D (theo thứ tự đó).
Đường tròn đường kính AC và BD cắt nhau tại ,
X Y . Đường thẳng XY cắt BC tại
Z . Lấy P là một điểm trên XY khác Z . Đường thẳng CP cắt đường tròn đường kính
AC tại điểm thứ hai là M và BP cắt đường tròn đường kính BD tại điểm thứ hai N .
Chứng minh rằng , ,
AM DN XY đồng quy.
T
N
A'
M
Ia
I O
B C
A
S
Trang 15
Lời giải
 Nếu P X
 thì ,
M X N X
  . Khi đó
, ,
AM DN XY đồng quy.
 Nếu P Y
 thì chứng minh tương tự trường
hợp trên.
 Nếu ,
X P Y
P  
  .
Gọi J AM XY
  , J DN XY
   .
Tứ giác JMCZ nội tiếp nên . .
PJ PZ PM PC
 .
Tương tự . .
PJ PZ PN PB
  .
Do P nằm trên trục đẳng phương của đường
tròn đường kính AC và đường tròn đường kính
BD nên . .
PM PC PN PB

. .
PJ PZ PJ PZ J J
 
    .
Vậy , ,
AM DN XY đồng quy.
3.5. [Luyện thi Quốc gia, Quốc tế của TP.Hồ Chí Minh]. Cho hai điểm ,
A B phân
biệt nằm trên đường tròn  
O và C nằm ngoài  
O . Gọi CS và CT là các tiếp tuyến
của C với  
O với ,
S T là các tiếp điểm, M là trung điểm của cung nhỏ AB . Các
đường thẳng ,
MS MT cắt AB lần lượt tại ,
E F . Đường thẳng đi qua ,
E F vuông góc
với AB cắt OS , OT lần lượt tại ,
X Y . Một đường thẳng bất kì qua C cắt  
O tại
,
P Q ( P nằm giữa C và Q ). Gọi R là giao của MP với AB , Z là tâm đường tròn
ngoại tiếp tam giác  
PQR . Chứng minh rằng , ,
X Y Z thẳng hàng.
Lời giải
Không mất tính tổng quát, ta giả sử E nằm
ngoài đoạn AB .
Ta thấy rằng EX //OM , mà OSM
 cân tại
O nên XES
 cân tại X . Xét đường tròn
( , )
X XE , dễ thấy rằng đường tròn này tiếp
xúc với  
O tại S và SC là tiếp tuyến
chung trong của hai đường tròn này.
Ta có:  
MAB MPA
 nên hai MAR
 và
MPA
 đồng dạng. Tương tự, MAE
 và
MSA
 cũng đồng dạng.
Từ đó suy ra ,
MA MR MA ME
MP MA MS MA
 
hay . .
MP MR MS ME
 .
Suy ra M nằm trên trục đẳng phương của
đường tròn ( )
X và ( )
PQR .
J M
N
Z
Y
X
A D
B C
P
Z
R
P
Y
X
F
E
M
T
S
O C
A
B
Q
Trang 16
Ta cũng có .
CS CP CQ

2
nên C cũng nằm trên trục đẳng phương của hai đường tròn
( )
X và ( )
PQR .
Từ đó suy ra CM là trục đẳng phương của ( )
X và ( )
PQR hay CM XZ
 .
Hoàn toàn tương tự, ta cũng có CM YZ
 . Vậy , ,
X Y Z thẳng hàng.
IV. Ứng dụng 4: Chứng minh một tập hợp điểm cùng nằm trên một đường tròn
4.1. [APMO 2009]. Trong cùng một mặt phẳng, cho ba đường tròn 1 2 3
, ,
   không đè
lên nhau và tiếp nhau ngoài nhau. Với mỗi điểm P trong mặt phẳng đó và nằm ngoài
cả ba vòng tròn, ta dựng sáu điểm 1 1 2 2 3 3
, , , , ,
A B A B A B theo cách sau đây:
Với mọi 1,2,3
i  các điểm ,
i i
A B phân biệt và cùng nằm trên i
 sao cho các đường
thẳng ,
i i
PA PB đều tiếp xúc với i
 . Ta gọi điểm P là điểm ngoại lệ nếu từ cách dựng
trên, ba đường thẳng 1 1 2 2 3 3
, ,
A B A B A B đồng quy. Chứng minh rằng mọi điểm ngoại lệ
nếu có cùng nằm trên một đường tròn.
Lời giải
Gọi i
O và i
r lần lượt là tâm và bán
kính của i
 với 1,2,3
i  .
Giả sử P là điểm ngoại lệ và ba
đường thẳng 1 1 2 2 3 3
, ,
A B A B A B đồng
quy tại Q . Dựng đường tròn đường
kính PQ . Gọi  là đường tròn tâm
O bán kính r . Ta sẽ chứng minh
mọi điểm ngoại lệ đều nằm trên  .
Giả sử 1 1 1 1
PO A B X
  .
Vì 1 1 1
PO A B
 nên 1
X nằm trên  .
Do 1
PA là một tiếp tuyến của 1
 nên
1 1
PAO
 vuông và đồng dạng với
1 1 1
A X O
 . Suy ra 1 1 1 1
1 1 1
O X O A
O A O P
 hay
2 2
1 1 1 1 1 1
.
O X O P O A r
  .
Mặt khác, 1 1 1
.
O X O P cũng là phương tích của điểm 1
O đối với  nên
  
2 2 2
1 1 1 1 1 1 1
.
r O X O P O O r O O r OO r
      và do đó
  
2 2 2
1 1 1 1 1 1
r OO r OO r OO r
     .
Như vậy 2
r chính là phương tích của điểm O đối với 1
 . Lý luận tương tự 2
r cũng là
phương tích của điểm O đối với đường tròn 2
 và 3
 . Suy ra O phải nằm trên tâm
đẳng phương của ba đường tròn đã cho.
Do r là căn bậc hai phương tích của điểm O đối với ba đường tròn đã cho nên r
không phụ thuộc vào vị trí điểm P. Điều này có nghĩa là mọi điểm ngoại lệ đều nằm
trên  .
r
X1
O
P
O1
A1
B1
Q
Trang 17
Nhận xét. Trong trường hợp tâm đẳng phương của ba đường tròn nằm tại vô cùng
(khi đó ba trục đẳng phương song song với nhau), sẽ không có điểm ngoại lệ nào nằm
trong mặt phẳng, điều này đã giải thích từ “nếu có” ở đề bài.
4.2. [IMO 2008]. Cho ABC
 nhọn với trực tâm H . Gọi 1 2 3
, ,
M M M lần lượt là trung
điểm , ,
BC CA AB . Đường tròn tâm 1
M bán kính 1
M H cắt BC tại 1 2
,
A A ; đường tròn
tâm 2
M bán kính 2
M H cắt AC tại 1 2
,
B B ; đường tròn tâm 3
M bán kính 3
M H cắt
AB tại 1 2
,
C C . Chứng minh rằng sáu điểm 1 2 1 2 1 2
, , , , ,
A A B B C C cùng thuộc một đường
tròn.
Lời giải
Do 1 2
M M // AB ( 1 2
M M đường trung bình) và
AB HC
 ( H trực tâm) nên 1 2
M M HC
 .
1 2 1 2
. .
CA CA CB CB
 
Bốn điểm 1 2 1 2
, , ,
A A B B thuộc đường tròn và
gọi đường tròn này là  
1
 .
Chứng minh tương tự ta thu được:
- Bốn điểm 1 2 1 2
, , ,
A A B B thuộc đường tròn  
2

- Bốn điểm 1 2 1 2
, , ,
C C B B thuộc đường tròn  
3

Nếu sáu điểm 1 2 1 2 1 2
, , , , ,
A A B B C C không cùng
thuộc một đường tròn thì các trục đẳng phương
của ba đường tròn      
1 2 3
, ,
   phải đồng
quy tại một điểm nhưng chúng lại cắt nhau tại
, ,
A B C nên đây là điều vô lý. Vậy mệnh đề phản chứng là sai, ta đi đến kết luận của
bài toán.
4.3.[IMO Shortlist 2006, Ukraine]. Cho hình thang ABCD với hai cạnh song song
AB CD
 . Hai điểm K và L nằm trên các đoạn thẳng AB và CD theo thứ tự thỏa
mãn
AK DL
KB LC
 . Giả sử rằng có các điểm P và Q nằm trên đoạn KL thỏa mãn
 
APB BCD
 và  
CQD ABC
 . Chứng minh rằng các điểm , , ,
P Q B C cùng thuộc một
đường tròn.
Lời giải
Từ giả thiết, ta có , ,
AK DL
AD BC KL
BK CL
  đồng quy tại E .
Dựng đường tròn  
1
O qua hai điểm ,
C D và tiếp xúc với BC ; dựng đường tròn  
2
O
qua hai điểm ,
A B và tiếp xúc với BC . Khi đó   
DQC ABC DCE
  nên  
1
Q O
 .
Chứng minh tương tự ta cũng được  
2
P O
 .
Gọi F là giao điểm thứ hai của EQ với  
1
O .
B2
B1
A2
A1
C2
C1
H
M2
M3
M1
A
B C
Trang 18
Ta có
2
.
EF EQ EC
 (1)
Mặt khác,  
1 2 2 1
O CD O BA AO B DO C
  
1
1 2
2
, ,
O C DC EC
k E O O
O B AB EB
     thẳng hàng
và 1
1 2
2
EO
k EO kEO
EO
  

 

.
Suy ra phép vị tự      
1 2
,
:
E k
V O O
 .
Mà , ,
E F P thẳng hàng,    
1 2
,
O
F P O
  dẫn
đến
EF EC
EF kEP k
EP EB
   

 

(2)
Từ (1) và (2) suy ra . .
EP EQ EC EB
 .
Vậy bốn điểm , , ,
P Q B C cùng thuộc một đường
tròn.
4.4. [International Zhautykov Olympiad 2008]. Trong mặt phẳng cho hai đường
tròn  
1
O và  
2
O . Gọi 1 2
A A là tiếp tuyến chung của hai đường tròn
   
 
1
1 2 2
,
A O A O
  , K là trung điểm 1 2
A A . Từ K lần lượt kẻ hai tiếp tuyến
1 2
,
KB KB đến    
1 2
,
O O . Gọi 1 1 2 2 1 2
,
L A B A B P KL O O
    . Chứng minh rằng
1 2
, , ,
B B P L cùng nằm trên một đường tròn.
Lời giải
Do 1 2 1 2
KA KA KB KB
   nên tứ giác
1 1 2 2
A B B A nội tiếp đường tròn
1 1 2 2
. .
LB LA LB LA
 
KL
 là trục đẳng phương của hai
đường tròn  
1
O và  
2
O
1 2
KL O O
  .
Để ý rằng ba điểm 1 1
, ,
A B P nhìn đoạn
1
O K dưới một góc 0
90 nên nội tiếp.
Tương tự, ta cũng chứng minh được tứ giác 2 2
A B PK nội tiếp.
Áp dụng định lý Miquel ta suy ra tứ giác 1 2
B PB L nội tiếp.
Vậy 1 2
, , ,
B B P L cùng nằm trên một đường tròn.
F
Q
P
L
E
A B
D C
K
O2
O1
P
L
K
A1
O1 O2
A2
B1 B2
Trang 19
V. Ứng dụng 5: Chứng minh bài toán điểm-đường cố định
5.1. [VMO 2014]. Cho ABC
 nhọn nội tiếp đường tròn  
O , trong đó ,
B C cố định
và A thay đổi trên  
O . Trên các tia AB và AC lần lượt lấy các điểm M và N sao
cho MA MC
 và NA NB
 . Các đường tròn ngoại tiếp các tam giác AMN và ABC
cắt nhau tại P  
P A

 . Đường thẳng MN cắt BC tại Q .
a) Chứng minh rằng ba điểm , ,
A P Q thẳng hàng.
b) Gọi D là trung điểm của BC . Các đường tròn có tâm là ,
M N và cùng đi qua A
cắt nhau tại K  
K A

 . Đường thẳng qua A vuông góc với AK cắt BC tại E .
Đường tròn ngoại tiếp ADE
 cắt  
O tại F  
F A

 . Chứng minh rằng AF đi qua
một điểm cố định.
Lời giải
a) Không mất tính tổng quát, ta giả sử
AB AC
 như hình vẽ, các trường
hợp còn loại hoàn toàn tương tự.
Khi đó, M nằm ngoài đoạn AB và
N nằm trong đoạn AC . Vì NA NB

nên  
NBA NAB
 ; lại vì MA MC

nên  
MCA MAC
 .
Suy ra  
NBA MCA
 hay tứ giác
BMCN nội tiếp và ta được:
. .
QM QN QB QC
 .
Từ đây suy ra Q có cùng phương tích
đến hai đường tròn  
O và  
AMN
nên nó nằm trên trục đẳng phương
AP của hai đường tròn này. Do đó
, ,
A P Q thẳng hàng.
b) Ta thấy rằng đường tròn  
ODC tiếp xúc với  
O tại C nên trục đẳng phương của
hai đường tròn này là tiếp tuyến d của  
O tại C .
Ta cần chứng minh  
O ADE
 . Thật vậy, ta có ,
O M cùng nằm trên trung trực của
AC nên OM AC
 . Tương tự, ON AB
 cho nên O là trực tâm AMN
 .
Suy ra AO MN
 .
Xét hai đường tròn    
, , ,
M MA N NA ta thấy AK MN
 . Suy ra , ,
A O K thẳng hàng
nên  0
90
OAE  . Mặt khác,  0
90
ODE  nên tứ giác AODE nội tiếp hay  
O ADE
 .
Do đó, trục đẳng phương của  
ADE và  
ODC chính là OD . Hơn nữa, trục đẳng
phương của  
O và  
ADE là AF .
d
I
F
E
Q
K
P
O
D
M
A
B
C
N
Trang 20
Nhận thấy ba đường tròn      
, ,
O ADE ODC đôi một có các trục đẳng phương là
, ,
OD d AF nên theo định lý 3 [về tâm đẳng phương] ba đường này đồng quy tại một
điểm. Vậy AF đi qua giao điểm I OD d
  và I là một điểm cố định.
5.2. [Chọn đội tuyển phổ thông năng khiếu (PTNK) 2008].
Cho tam giác ABC có đỉnh A cố định và ,
B C thay đổi trên đường thẳng d cố định
sao cho nếu gọi A là hình chiếu của A lên d thì .
A B A C
  âm và không đổi. Gọi M
là hình chiếu của A lên AB . Gọi N là hình chiếu của A lên AC . Gọi K là giao
điểm của các tiếp tuyến của đường tròn ngoại tiếp A MN

 tại M và N . Chứng minh
rằng K thuộc đường thẳng cố định.
Lời giải
Ta có
2
. .
AM AB AA AN AC

 
tứ giác BMNC nội tiếp
 
AMN ACB
  .
Lại có  
ADB ACB
 nên  
AMN ADB
 .
Điều này dẫn đến tứ giác MPDB nội tiếp.
Do đó
2
. .
AP AD AM AB AA
  .
Mà , ,
A A D
 cố định nên P cố định.
Gọi H là hình chiếu của K trên AA.
Khi đó 2 2
1
. .
4
OP OH OI OK ON AA
   .
Mà , , ,
A P A O
 cố định nên H cố định.
Vậy K thuộc đường thẳng qua H và vuông góc với AA.
5.3. [VMO 2003]. Cho đường tròn  
1 1
;
O R tiếp xúc ngoài với đường tròn  
2 2
;
O R tại
M với 2 1
R R
 . Xét điểm A di động trên đường tròn  
2 2
;
O R sao cho 1 2
, ,
A O O
không thẳng hàng. Từ A kẻ tiếp tuyến ,
AB AC đến  
1
O . Các đường thẳng ,
MB MC
cắt  
2
O tại ,
E F . Gọi D là giao điểm của EF với tiếp tuyến tại A của đường tròn
 
2
O . Chứng minh rằng D di động trên một đường thẳng cố định.
Lời giải
Qua M kẻ tiếp tuyến chung của  
1
O và  
2
O .
Ta có    
MCA CMy FMD FAM
  
FAM FCA
  (g.g)
2 2 2
1 1
.
FA FM FC FO R
    (1)
Tương tự, 2 2 2
1 1
EA EO R
  (2)
H
P
D
I
K
O
N
M
A'
A
B C
y
A
M
O2
O1
B
C
E
F
D
Trang 21
Ký hiệu  
;0
A là đường tròn tâm A, bán kính 0
R  . Từ (1) và (2) ta suy ra EF là
trục đẳng phương của  
;0
A với  
1
O .
Mà D nằm trên EF nên    
1 2
2 2 2
1 1 / /
D O D O
DA DO R P P
    .
Vậy D nằm trên trục đẳng phương của hai đường tròn cố định  
1
O và  
2
O .
5.4.[Đề đề nghị kỳ thi Olympic duyên hải và đồng bằng Bắc Bộ năm 2014].
Cho ABC
 vuông ở A, đường cao AH . Gọi ,
E F theo thứ tự là hình chiếu của H
trên ,
AB AC . Chứng minh rằng: Khi ,
A H không thay đổi còn ,
B C thay đổi thì
đường tròn ngoại tiếp tứ giác BCFE luôn đi qua 2 điểm cố định.
Lời giải
Theo giả thiết, trong các tam giác vuông AHB và
AHC ta có:
2
.
AE AB AH
 và
2
.
AF AC AH
 .
Do đó: . .
AE AB AF AC
 . Suy ra tứ giác BECF nội
tiếp.
Gọi ,
P Q là giao điểm của AH với đường tròn
 
BEFC .
Do tứ giác BPCQ nội tiếp nên: . .
HP HQ HB HC
 .
Trong tam giác vuông ABC có: 2
.
HB HC AH
 .
Do đó 2
. .
HP HQ HB HC AH
  .
Mặt khác: 2
. .
AP AQ AE AB AH
  .
Suy ra
2
( )( )
AH HP AH HQ AH
  
2 2
( ) .
.
AH AH HQ HP HP HQ AH
AH HQ HP
    
  
Do đó: 2
.
HP HQ AH
HP HQ AH
 






.
Giải hệ này ta được:
5 1
2
5 1
2
HP AH
HQ AH
 





 


 ,
P Q cố định.
Vậy đường tròn  
BEFC luôn đi qua 2 điểm cố định ,
P Q.
P
Q
E
F
H
A
B C
Trang 22
VI. Ứng dụng 6: Tính toán các đại lượng hình học
6.1. [USA 1998]. Cho hai đường tròn đồng tâm O ký hiệu là  
1
C và  
2
C ( 
2
C nằm
trong  
1
C ). Từ một điểm A nằm trên  
1
C kẻ tiếp tuyến AB đến  
2
C . AB cắt  
2
C
lần thứ hai tại C . D là trung điểm của AB . Một đường thẳng qua A cắt  
2
O tại
,
E F sao cho đường trung trực của DF và EC giao nhau tại điểm M nằm trên AC .
Tính tỉ số
AM
MC
.
Lời giải
Rõ ràng B là trung điểm AC .
Ta có  
2
2
/
1
. .2 .
2
A C
AE AF AB AB AB AD AC
P     .
Suy ra tứ giác DCFE nội tiếp đường tròn. Do đó M là
tâm đường tròn ngoại tiếp tứ giác DCFE .
Mà M nằm trên AC nên
1
2
MD MC DC
  .
Điều đó giúp ta tính được
5
3
4
3 5
4
AM AB
AM
MC
MC AB


 


  


 




.
6.2. [Russian MO 2008]. Cho ABC
 nội tiếp đường tròn  
;
O R , ngoại tiếp đường
tròn  
;
I r . Đường tròn  
I tiếp xác với ,
AB AC lần lượt tại ,
X Y . Gọi K là điểm
chính giữa cung AB không chứa C . Giả sử XY chia đôi đoạn AK . Tính 
BAC ?
Lời giải
Gọi S XY AK
  .
Ta có  
KAB KCB
 ;
  
0
180
SXA AXY BIC AXS CIB
    
.
AS AX AX CB
AS
CB CI CI
   
1 .
2
AX BC
AK
CI
  hay . 2 .
KI CI AX BC
 .
Đặt
, , , ,
2
ABC
a b c
BC a CA b AB c S S p
 
    

Ta có       2 2 2
2 2 . 4
4
abc S
Rr p a a p a a bc p p a b c bc a
S p
           
M
E
D C
A
O
B
F
I
O
B
A
C
X
Y
K S
Trang 23
Mặt khác, theo định lý hàm côsin ta có 2 2 2
2 cos
a b c bc A
   .
Khi đó 2 2 2 2 0
1
2 cos cos 120
2
b c bc b c bc A A A
         .
Vậy  0
120
BAC  .
6.3. [Russian MO 2007]. Cho hai đường tròn  
1
O và  
2
O cắt nhau tại ,
A B . Gọi
,
PQ RS là hai tiếp tuyến chung của hai đường tròn    
 
1 2
; ,
, O
P R O Q S
  . Giả sử
RB// PQ , RB cắt  
2
O lần nữa tại W . Tính tỉ số
RB
BW
?
Lời giải
Gọi J AB PQ
  . Suy ra J là trung
điểm của đoạn thẳng PQ .
Rõ ràng AB // PR; PQ // RB nên tứ giác
PJBR là hình bình hành
PJ RB
 
2 2 2 2
. 4 4
RB RW RS PQ PJ RB
    
4
RW RB
  .
Vậy
1
3
RB
RW
 .
VII. Ứng dụng 7: Chứng minh một số mệnh đề về góc của tam giác và tìm quỹ tích
điểm.
7.1. [Belarus 2000, problem 1]. Cho tứ giác ABCD có hai đường chéo cắt nhau ở
M . Đường phân giác của 
ACD cắt tia BA ở K . Chứng minh rằng:
nếu . . .
MA MC MACD MB MD
  thì  
BKC CDB
 .
Lời giải
Gọi N CK BD
  . Áp dụng định lý về đường phân giác cho
MCD
 ta có:
.
CD MC MC DN
CD
DN MN MN
   .
Khi đó:
.
. . .
MC DN
MB MD MA MC MA
MN
 
   
. 1 . .
DN MD
MA MC MA MC
MN MN
 
  
 
 
hay . .
MA MC MB MN
 .
S
Q
B
A
O1
O2
P
R
J
W
N
K
M
A
D
C
B
Trang 24
Vì M nằm trong tứ giác ABCN , theo định lý về phương tích thì , , ,
A B C N cùng
nằm trên đường tròn.
Từ đó     
KBD ABN ACN NCD KCD
    . Suy ra , , ,
K B C D cùng nằm trên một
đường tròn. Do đó ta có  
BKC CDB
 .
7.2.[Belarus 2000, problem 2]. Cho ABC
 và 
2
C

 . Gọi M là trung điểm của
cạnh huyền AB , H là chân đường cao CH và P là điểm nằm trong tam giác sao cho
AP AC
 . Chứng minh rằng PM là phân giác 
.BPH khi và chi 
3
A

 .
Lời giải
Điểm P nằm trên đường tròn
tâm A bán kính AC .
Đường tròn  cắt các đường
thẳng ,
CH MH và PH tương
ứng tại ,
D N và Q .
Vì MA MC
 ,  0
60
A  khi và
chỉ khi ACM
 đều. Nghĩa là khi
và chỉ khi M N
 .
Điều đó khẳng định PM là phân
giác 
BPH khi và chỉ khi
M N
 .
Thật vậy, AH là đường cao
thuộc đáy của tam giác cân
ACD nên H là trung điểm của CD và CD là một dây cung của đường tròn  . Theo
định lý về phương tích của một điểm ta có 2
. .
PH HQ CH HD CH
  . Vì CH là
đường cao thuộc cạnh huyền của ABC
 nên 2
.
CH AH HB
 . Như vậy
. .
PH HQ AH HB
 .
Do H AB PQ
  nên tứ giác APBQ nội tiếp. Để ý trên đường tròn  rằng:
   
2 2
QAB QAN QPN HPN
   .
Thế thì    
2
HPB QPB QAB HPN
   .
Vì N HB PN
  (PN là phân giác 
HPB ). Do đó PM là phân giác 
HPB khi và chỉ
khi M N
 .
7.3. [APMO 1995]. Cho tứ giác lồi cố định ABCD nội tiếp trong đường tròn mà AB
không song song với CD . Tìm quỹ tích các điểm P thỏa mãn điều kiện: ta có thể tìm
được hai đường tròn lần lượt nhận AB và CD làm dây cung sao cho chúng tiếp xúc
nhau tại P .
Q
N
D
M
H
A
C
B
P
Trang 25
Lời giải
Giả sử AB CD X
  .
Từ X kẻ một tiếp tuyến đến đường tròn
ngoại tiếp tứ giác ABCD và gọi R là độ
dài đoạn tiếp tuyến này. Như vậy quỹ tích
cần tìm là đường tròn tâm X , bán kính R.
Thật vậy, ta gọi  
0
C là đường tròn tâm X
bán kính R.
Lấy một điểm P bất kì trên  
0
C . Xét
đường tròn ngoại tiếp tứ giác ABCD ta có:
2
. .
R XA XB XC XD
  .
Suy ra 2
. .
XP XA XB XC XD
  .
Gọi  
1
C là đường tròn đi qua ba điểm
, ,
C D P , ta có 2
.
XC XD XP
 nên XP là tiếp tuyến của đường tròn  
1
C .
Tương tự gọi  
2
C là đường tròn đi qua ba điểm , ,
A B P thì XP là tiếp tuyến của
đường tròn này.
Như vậy  
1
C và  
2
C tiếp xúc nhau tại P.
Ta chú ý rằng nếu P là một trong bốn giao điểm của AB hoặc CD với  
0
C thì
không thể dựng các đường tròn như trên được, trừ khi ta chấp nhận rằng đó là đường
tròn suy biến thành đường thẳng.
Như vậy ta đã chứng minh được rằng tất cả các điểm nằm trên  
0
C đều là các điểm
của quỹ tích (hoặc trừ đi bốn điểm nói trên nếu không chấp nhận trường hợp suy biến).
Bây giờ đảo lại, ta sẽ chứng minh rằng không có điểm nào của quỹ tích mà không nằm
trên  
0
C .
Thật vậy, với bất kì đường tròn nào đi qua hai điểm ,
C D, chỉ có hai đường tròn qua
,
A B và tiếp xúc với nó (điều này rõ ràng nếu ta xét cách mà đường tròn qua ,
A B thay
đổi khi tâm của nó di chuyển trên đường trung trực của AB ), do vậy có nhiều nhất là
hai điểm thuộc quỹ tích nằm trên đường tròn đi qua ,
C D cho trước.
Mà đây là hai giao điểm của đường tròn đó với  
0
C , vậy không có điểm nào của quỹ
tích mà không nằm trên  
0
C .
X
A
B
C
D
Trang 26
C. BÀI TẬP ĐỀ NGHỊ
1. Cho đường tròn  
O tiếp xúc đường thẳng d tại H . Hai điểm ,
M N di động trên d
sao cho 2
.  
HM HN k ( 0

k cho trước ). Từ ,
M N kẻ tiếp tuyến MA và NB của
 
O
(với , 

A B H ). Chứng minh rằng:
a) Đường tròn  
OMN luôn đi qua 2 điểm cố định.
b) Đường thẳng AB luôn đi qua 1 điểm cố định.
2. Cho AB và AC là các tiếp tuyến của đường tròn  
O với ,
B C thuộc  
O . Lấy
điểm M bất kì trên AC ( ,
M A khác phía so với C ). Giả sử (O) cắt đường tròn
(ABM) tại điểm thứ hai ,
P Q là chân đường vuông góc hạ từ C xuống MB . Chứng
minh rằng:  
2
MPQ AMB
 .
3. Cho tam giác ABC có đường tròn tâm I nội tiếp, tiếp xúc các cạnh , ,
BC CA AB tại
, ,
D E F . AI cắt đường tròn  
I tại ,
M N (M nằm giữa A và N ). DM cắt cạnh EF
tại K , NK cắt đường tròn  
I tại điểm P khác N . Chứng minh rằng các điểm
, ,
A P D thẳng hàng.
4. Cho tứ giác ABCD nội tiếp đường tròn tâm O bán kính R. Các đường thẳng AB
và CD cắt nhau tại P, đường thẳng AD và đường thẳng BC cắt nhau tại Q . Chứng
minh rằng: 
2
2
cos
R
POQ
OP OQ
 
  

 
.
5. Cho tứ giác ABCD nội tiếp đường tròn  
;
O R . Gọi , ,
P Q M lần lượt là giao điểm
của các cặp đường thẳng AB và DC , AD và BC , AC và BD . Chứng minh rằng bán
kính đường tròn ngoại tiếp các tam giác OPQ,OMP và OMQ bằng nhau.
6. Cho đường tròn tâm O đường kính AB . Một điểm H thuộc đoạn AB . Đường
thẳng qua H cắt đường tròn tại C . Đường tròn đường kính CH cắt AC , BC và
 
O lần lượt tại ,
D E và F .
a) Chứng minh rằng ,
AB DE và CF đồng quy.
b) Đường tròn  
;
C CH cắt  
O tại P và Q . Chứng minh rằng , , ,
P D E Q thẳng
hàng.
7. Cho tam giác ABC , D thuộc đoạn BC , E thuộc đoạn AD . Đường tròn ngoại tiếp
BDE
 cắt AB tại K . Đường tròn ngoại tiếp CDE
 cắt AC tại L . Gọi M là giao
điểm của DK với BE , N là giao điểm của DL với CE , O là tâm đường tròn ngoại
tiếp tam giác EBC . Chứng minh rằng AO MN
 .
===HẾT===

More Related Content

What's hot

Cực trị hình học cực hay nè,,,
Cực trị hình học cực hay nè,,,Cực trị hình học cực hay nè,,,
Cực trị hình học cực hay nè,,,
bichshiho20
 
Hệ Hoán Vị Vòng Quanh
Hệ Hoán Vị Vòng QuanhHệ Hoán Vị Vòng Quanh
Hệ Hoán Vị Vòng Quanh
Nhập Vân Long
 
Luận văn: Các bài toán về hệ thức lượng trong tam giác, HOT, 9đ
Luận văn: Các bài toán về hệ thức lượng trong tam giác, HOT, 9đLuận văn: Các bài toán về hệ thức lượng trong tam giác, HOT, 9đ
Luận văn: Các bài toán về hệ thức lượng trong tam giác, HOT, 9đ
Dịch vụ viết bài trọn gói ZALO 0917193864
 
Bài toán số học liên quan tới lũy thữa
Bài toán số học liên quan tới lũy thữaBài toán số học liên quan tới lũy thữa
Bài toán số học liên quan tới lũy thữaThế Giới Tinh Hoa
 
9 phương pháp giải phương trình nghiệm nguyên
9 phương pháp giải phương trình nghiệm nguyên9 phương pháp giải phương trình nghiệm nguyên
9 phương pháp giải phương trình nghiệm nguyênThấy Tên Tao Không
 
19 phương phap chứng minh bất đẳng thức
19 phương phap chứng minh bất đẳng thức19 phương phap chứng minh bất đẳng thức
19 phương phap chứng minh bất đẳng thứcThế Giới Tinh Hoa
 
Chuyên đề tìm giới hạn nâng cao
Chuyên đề tìm giới hạn nâng caoChuyên đề tìm giới hạn nâng cao
Chuyên đề tìm giới hạn nâng cao
Bống Bình Boong
 
CHUYÊN ĐỀ HÌNH HỌC ÔN THI VÀO LỚP 10 CÁC TRƯỜNG CHUYÊN
CHUYÊN ĐỀ HÌNH HỌC ÔN THI VÀO LỚP 10 CÁC TRƯỜNG CHUYÊNCHUYÊN ĐỀ HÌNH HỌC ÔN THI VÀO LỚP 10 CÁC TRƯỜNG CHUYÊN
CHUYÊN ĐỀ HÌNH HỌC ÔN THI VÀO LỚP 10 CÁC TRƯỜNG CHUYÊN
BOIDUONGTOAN.COM
 
Các phương pháp hay giải Phuong trinh-vo-ty
Các phương pháp hay giải Phuong trinh-vo-tyCác phương pháp hay giải Phuong trinh-vo-ty
Các phương pháp hay giải Phuong trinh-vo-tyroggerbob
 
Sử Dụng Hàng Điểm Điều Hòa Trong Giải Toán Hình Học Phẳng.doc
Sử Dụng Hàng Điểm Điều Hòa Trong Giải Toán Hình Học Phẳng.docSử Dụng Hàng Điểm Điều Hòa Trong Giải Toán Hình Học Phẳng.doc
Sử Dụng Hàng Điểm Điều Hòa Trong Giải Toán Hình Học Phẳng.doc
DV Viết Luận văn luanvanmaster.com ZALO 0973287149
 
Tuyển tập chuyên đề bất đẳng thức có lời giải chi tiết 2
Tuyển tập chuyên đề bất đẳng thức có lời giải chi tiết 2Tuyển tập chuyên đề bất đẳng thức có lời giải chi tiết 2
Tuyển tập chuyên đề bất đẳng thức có lời giải chi tiết 2
https://www.facebook.com/garmentspace
 
đại số tuyến tính 2 ( không gian eculid )
đại số tuyến tính 2 ( không gian eculid )đại số tuyến tính 2 ( không gian eculid )
đại số tuyến tính 2 ( không gian eculid )
Bui Loi
 
Chuyên đề Đẳng Thức và Bất đẳng thức - Bồi dưỡng HSG môn Toán lớp 9
Chuyên đề Đẳng Thức và Bất đẳng thức - Bồi dưỡng HSG môn Toán lớp 9Chuyên đề Đẳng Thức và Bất đẳng thức - Bồi dưỡng HSG môn Toán lớp 9
Chuyên đề Đẳng Thức và Bất đẳng thức - Bồi dưỡng HSG môn Toán lớp 9
BOIDUONGTOAN.COM
 
Chuyen de toan logic roi rac li thuyet to hop
Chuyen de toan logic  roi rac li thuyet to hopChuyen de toan logic  roi rac li thuyet to hop
Chuyen de toan logic roi rac li thuyet to hoplephucduc06011999
 
Luận văn: Một số phương pháp giải phương trình hàm, HOT, 9đ
Luận văn: Một số phương pháp giải phương trình hàm, HOT, 9đLuận văn: Một số phương pháp giải phương trình hàm, HOT, 9đ
Luận văn: Một số phương pháp giải phương trình hàm, HOT, 9đ
Dịch vụ viết bài trọn gói ZALO: 0909232620
 
Tổng hợp các phương pháp giải phương trình và hệ phương trình môn toán
Tổng hợp các phương pháp giải phương trình và hệ phương trình môn toánTổng hợp các phương pháp giải phương trình và hệ phương trình môn toán
Tổng hợp các phương pháp giải phương trình và hệ phương trình môn toán
huyenltv274
 
[Phần 2] Tuyển tập 35 công thức giải nhanh bài tập Hóa học vô cơ - Megabook.vn
[Phần 2] Tuyển tập 35 công thức giải nhanh bài tập Hóa học vô cơ - Megabook.vn[Phần 2] Tuyển tập 35 công thức giải nhanh bài tập Hóa học vô cơ - Megabook.vn
[Phần 2] Tuyển tập 35 công thức giải nhanh bài tập Hóa học vô cơ - Megabook.vn
Megabook
 

What's hot (20)

Bất đẳng thức hình học
Bất đẳng thức hình họcBất đẳng thức hình học
Bất đẳng thức hình học
 
Cực trị hình học cực hay nè,,,
Cực trị hình học cực hay nè,,,Cực trị hình học cực hay nè,,,
Cực trị hình học cực hay nè,,,
 
Hệ Hoán Vị Vòng Quanh
Hệ Hoán Vị Vòng QuanhHệ Hoán Vị Vòng Quanh
Hệ Hoán Vị Vòng Quanh
 
Luận văn: Các bài toán về hệ thức lượng trong tam giác, HOT, 9đ
Luận văn: Các bài toán về hệ thức lượng trong tam giác, HOT, 9đLuận văn: Các bài toán về hệ thức lượng trong tam giác, HOT, 9đ
Luận văn: Các bài toán về hệ thức lượng trong tam giác, HOT, 9đ
 
Bài toán số học liên quan tới lũy thữa
Bài toán số học liên quan tới lũy thữaBài toán số học liên quan tới lũy thữa
Bài toán số học liên quan tới lũy thữa
 
9 phương pháp giải phương trình nghiệm nguyên
9 phương pháp giải phương trình nghiệm nguyên9 phương pháp giải phương trình nghiệm nguyên
9 phương pháp giải phương trình nghiệm nguyên
 
19 phương phap chứng minh bất đẳng thức
19 phương phap chứng minh bất đẳng thức19 phương phap chứng minh bất đẳng thức
19 phương phap chứng minh bất đẳng thức
 
Chuyên đề tìm giới hạn nâng cao
Chuyên đề tìm giới hạn nâng caoChuyên đề tìm giới hạn nâng cao
Chuyên đề tìm giới hạn nâng cao
 
CHUYÊN ĐỀ HÌNH HỌC ÔN THI VÀO LỚP 10 CÁC TRƯỜNG CHUYÊN
CHUYÊN ĐỀ HÌNH HỌC ÔN THI VÀO LỚP 10 CÁC TRƯỜNG CHUYÊNCHUYÊN ĐỀ HÌNH HỌC ÔN THI VÀO LỚP 10 CÁC TRƯỜNG CHUYÊN
CHUYÊN ĐỀ HÌNH HỌC ÔN THI VÀO LỚP 10 CÁC TRƯỜNG CHUYÊN
 
Các phương pháp hay giải Phuong trinh-vo-ty
Các phương pháp hay giải Phuong trinh-vo-tyCác phương pháp hay giải Phuong trinh-vo-ty
Các phương pháp hay giải Phuong trinh-vo-ty
 
Sử Dụng Hàng Điểm Điều Hòa Trong Giải Toán Hình Học Phẳng.doc
Sử Dụng Hàng Điểm Điều Hòa Trong Giải Toán Hình Học Phẳng.docSử Dụng Hàng Điểm Điều Hòa Trong Giải Toán Hình Học Phẳng.doc
Sử Dụng Hàng Điểm Điều Hòa Trong Giải Toán Hình Học Phẳng.doc
 
Phương trình hàm đa thức
Phương trình hàm đa thứcPhương trình hàm đa thức
Phương trình hàm đa thức
 
Tuyển tập chuyên đề bất đẳng thức có lời giải chi tiết 2
Tuyển tập chuyên đề bất đẳng thức có lời giải chi tiết 2Tuyển tập chuyên đề bất đẳng thức có lời giải chi tiết 2
Tuyển tập chuyên đề bất đẳng thức có lời giải chi tiết 2
 
đại số tuyến tính 2 ( không gian eculid )
đại số tuyến tính 2 ( không gian eculid )đại số tuyến tính 2 ( không gian eculid )
đại số tuyến tính 2 ( không gian eculid )
 
Chuyên đề Đẳng Thức và Bất đẳng thức - Bồi dưỡng HSG môn Toán lớp 9
Chuyên đề Đẳng Thức và Bất đẳng thức - Bồi dưỡng HSG môn Toán lớp 9Chuyên đề Đẳng Thức và Bất đẳng thức - Bồi dưỡng HSG môn Toán lớp 9
Chuyên đề Đẳng Thức và Bất đẳng thức - Bồi dưỡng HSG môn Toán lớp 9
 
Chuyen de toan logic roi rac li thuyet to hop
Chuyen de toan logic  roi rac li thuyet to hopChuyen de toan logic  roi rac li thuyet to hop
Chuyen de toan logic roi rac li thuyet to hop
 
Luận văn: Một số phương pháp giải phương trình hàm, HOT, 9đ
Luận văn: Một số phương pháp giải phương trình hàm, HOT, 9đLuận văn: Một số phương pháp giải phương trình hàm, HOT, 9đ
Luận văn: Một số phương pháp giải phương trình hàm, HOT, 9đ
 
Tổng hợp các phương pháp giải phương trình và hệ phương trình môn toán
Tổng hợp các phương pháp giải phương trình và hệ phương trình môn toánTổng hợp các phương pháp giải phương trình và hệ phương trình môn toán
Tổng hợp các phương pháp giải phương trình và hệ phương trình môn toán
 
Tu dien-vuong-tinh-chat
Tu dien-vuong-tinh-chatTu dien-vuong-tinh-chat
Tu dien-vuong-tinh-chat
 
[Phần 2] Tuyển tập 35 công thức giải nhanh bài tập Hóa học vô cơ - Megabook.vn
[Phần 2] Tuyển tập 35 công thức giải nhanh bài tập Hóa học vô cơ - Megabook.vn[Phần 2] Tuyển tập 35 công thức giải nhanh bài tập Hóa học vô cơ - Megabook.vn
[Phần 2] Tuyển tập 35 công thức giải nhanh bài tập Hóa học vô cơ - Megabook.vn
 

Similar to Ứng dụng phương tích và trục đẳng phương vào bài toán hình học phẳng

Chuyên đề phương tích và ứng dụng
Chuyên đề phương tích và ứng dụngChuyên đề phương tích và ứng dụng
Chuyên đề phương tích và ứng dụng
Ngo Quang Viet
 
Chuyên đề phương tích và ứng dụng
Chuyên đề phương tích và ứng dụngChuyên đề phương tích và ứng dụng
Chuyên đề phương tích và ứng dụng
Vui Lên Bạn Nhé
 
Chuyen de-9-hinh-hoc-phang
Chuyen de-9-hinh-hoc-phangChuyen de-9-hinh-hoc-phang
Chuyen de-9-hinh-hoc-phanggadaubac2003
 
ôn hình lớp 9
ôn hình lớp 9ôn hình lớp 9
ôn hình lớp 9
tamhvtc
 
Phep nghich dao __
Phep nghich dao  __Phep nghich dao  __
Phep nghich dao __Duc Tung
 
Vi tri tuong doi cua hai duong tron
Vi tri tuong doi cua hai duong tronVi tri tuong doi cua hai duong tron
Vi tri tuong doi cua hai duong tron
hoaadc2016
 
PHÉP NHÂN MỘT SỐ VỚI MỘT VECTƠ
PHÉP NHÂN MỘT SỐ VỚI MỘT VECTƠPHÉP NHÂN MỘT SỐ VỚI MỘT VECTƠ
PHÉP NHÂN MỘT SỐ VỚI MỘT VECTƠ
DANAMATH
 
9 tu giac noi tiep
9 tu giac noi tiep9 tu giac noi tiep
9 tu giac noi tiep
Hồng Quang
 
Chuyen de-hinh-hoc-lop-9-hay-chuyen-de-hinh-hoc-lop-9-hay-tong-hop-on-tap-hin...
Chuyen de-hinh-hoc-lop-9-hay-chuyen-de-hinh-hoc-lop-9-hay-tong-hop-on-tap-hin...Chuyen de-hinh-hoc-lop-9-hay-chuyen-de-hinh-hoc-lop-9-hay-tong-hop-on-tap-hin...
Chuyen de-hinh-hoc-lop-9-hay-chuyen-de-hinh-hoc-lop-9-hay-tong-hop-on-tap-hin...
Toán THCS
 
Chuyen de hinh hoc 9
Chuyen de hinh hoc 9Chuyen de hinh hoc 9
Chuyen de hinh hoc 9
Nguyễn Hoàng Phước
 
840693 474666 thay buu_tong hop hinh hoc
840693 474666 thay buu_tong hop hinh hoc840693 474666 thay buu_tong hop hinh hoc
840693 474666 thay buu_tong hop hinh hochoang ngoc quang
 
CHUYÊN ĐỀ :TỌA ĐỘ PHẲNG - PHƯƠNG PHÁP VECTƠ
CHUYÊN ĐỀ :TỌA ĐỘ PHẲNG - PHƯƠNG PHÁP VECTƠCHUYÊN ĐỀ :TỌA ĐỘ PHẲNG - PHƯƠNG PHÁP VECTƠ
CHUYÊN ĐỀ :TỌA ĐỘ PHẲNG - PHƯƠNG PHÁP VECTƠ
DANAMATH
 
Bo de hinh hoc thcs
Bo de hinh hoc thcsBo de hinh hoc thcs
Bo de hinh hoc thcs
khanh271295
 
Toan pt.de126.2011
Toan pt.de126.2011Toan pt.de126.2011
Toan pt.de126.2011BẢO Hí
 
chuyen de hinh hoc vao 10 (full) ttc
 chuyen de hinh hoc vao 10 (full) ttc chuyen de hinh hoc vao 10 (full) ttc
chuyen de hinh hoc vao 10 (full) ttc
Toán THCS
 
Tập 3 chuyên đề Toán học: Hình học phẳng Oxy - Megabook.vn
Tập 3 chuyên đề Toán học: Hình học phẳng Oxy - Megabook.vnTập 3 chuyên đề Toán học: Hình học phẳng Oxy - Megabook.vn
Tập 3 chuyên đề Toán học: Hình học phẳng Oxy - Megabook.vn
Megabook
 
THAM SỐ HÓA TRONG TỌA ĐỘ PHẲNG
THAM SỐ HÓA TRONG TỌA ĐỘ PHẲNGTHAM SỐ HÓA TRONG TỌA ĐỘ PHẲNG
THAM SỐ HÓA TRONG TỌA ĐỘ PHẲNG
DANAMATH
 
Chuyên đề về đường tròn
Chuyên đề về đường trònChuyên đề về đường tròn
Chuyên đề về đường tròn
Ngo Quang Viet
 
các bài toán hình học lớp 9 có lời giải
các bài toán hình học lớp 9 có lời giảicác bài toán hình học lớp 9 có lời giải
các bài toán hình học lớp 9 có lời giải
Khoảnh Khắc Bình Yên
 
CHUYÊN ĐỀ QUAN HỆ VUÔNG GÓC - CHƯƠNG 3 HÌNH HỌC 11 (THẦYHOÀNG THÁI VIỆT)
CHUYÊN ĐỀ QUAN HỆ VUÔNG GÓC - CHƯƠNG 3 HÌNH HỌC 11 (THẦYHOÀNG THÁI VIỆT)CHUYÊN ĐỀ QUAN HỆ VUÔNG GÓC - CHƯƠNG 3 HÌNH HỌC 11 (THẦYHOÀNG THÁI VIỆT)
CHUYÊN ĐỀ QUAN HỆ VUÔNG GÓC - CHƯƠNG 3 HÌNH HỌC 11 (THẦYHOÀNG THÁI VIỆT)
Hoàng Thái Việt
 

Similar to Ứng dụng phương tích và trục đẳng phương vào bài toán hình học phẳng (20)

Chuyên đề phương tích và ứng dụng
Chuyên đề phương tích và ứng dụngChuyên đề phương tích và ứng dụng
Chuyên đề phương tích và ứng dụng
 
Chuyên đề phương tích và ứng dụng
Chuyên đề phương tích và ứng dụngChuyên đề phương tích và ứng dụng
Chuyên đề phương tích và ứng dụng
 
Chuyen de-9-hinh-hoc-phang
Chuyen de-9-hinh-hoc-phangChuyen de-9-hinh-hoc-phang
Chuyen de-9-hinh-hoc-phang
 
ôn hình lớp 9
ôn hình lớp 9ôn hình lớp 9
ôn hình lớp 9
 
Phep nghich dao __
Phep nghich dao  __Phep nghich dao  __
Phep nghich dao __
 
Vi tri tuong doi cua hai duong tron
Vi tri tuong doi cua hai duong tronVi tri tuong doi cua hai duong tron
Vi tri tuong doi cua hai duong tron
 
PHÉP NHÂN MỘT SỐ VỚI MỘT VECTƠ
PHÉP NHÂN MỘT SỐ VỚI MỘT VECTƠPHÉP NHÂN MỘT SỐ VỚI MỘT VECTƠ
PHÉP NHÂN MỘT SỐ VỚI MỘT VECTƠ
 
9 tu giac noi tiep
9 tu giac noi tiep9 tu giac noi tiep
9 tu giac noi tiep
 
Chuyen de-hinh-hoc-lop-9-hay-chuyen-de-hinh-hoc-lop-9-hay-tong-hop-on-tap-hin...
Chuyen de-hinh-hoc-lop-9-hay-chuyen-de-hinh-hoc-lop-9-hay-tong-hop-on-tap-hin...Chuyen de-hinh-hoc-lop-9-hay-chuyen-de-hinh-hoc-lop-9-hay-tong-hop-on-tap-hin...
Chuyen de-hinh-hoc-lop-9-hay-chuyen-de-hinh-hoc-lop-9-hay-tong-hop-on-tap-hin...
 
Chuyen de hinh hoc 9
Chuyen de hinh hoc 9Chuyen de hinh hoc 9
Chuyen de hinh hoc 9
 
840693 474666 thay buu_tong hop hinh hoc
840693 474666 thay buu_tong hop hinh hoc840693 474666 thay buu_tong hop hinh hoc
840693 474666 thay buu_tong hop hinh hoc
 
CHUYÊN ĐỀ :TỌA ĐỘ PHẲNG - PHƯƠNG PHÁP VECTƠ
CHUYÊN ĐỀ :TỌA ĐỘ PHẲNG - PHƯƠNG PHÁP VECTƠCHUYÊN ĐỀ :TỌA ĐỘ PHẲNG - PHƯƠNG PHÁP VECTƠ
CHUYÊN ĐỀ :TỌA ĐỘ PHẲNG - PHƯƠNG PHÁP VECTƠ
 
Bo de hinh hoc thcs
Bo de hinh hoc thcsBo de hinh hoc thcs
Bo de hinh hoc thcs
 
Toan pt.de126.2011
Toan pt.de126.2011Toan pt.de126.2011
Toan pt.de126.2011
 
chuyen de hinh hoc vao 10 (full) ttc
 chuyen de hinh hoc vao 10 (full) ttc chuyen de hinh hoc vao 10 (full) ttc
chuyen de hinh hoc vao 10 (full) ttc
 
Tập 3 chuyên đề Toán học: Hình học phẳng Oxy - Megabook.vn
Tập 3 chuyên đề Toán học: Hình học phẳng Oxy - Megabook.vnTập 3 chuyên đề Toán học: Hình học phẳng Oxy - Megabook.vn
Tập 3 chuyên đề Toán học: Hình học phẳng Oxy - Megabook.vn
 
THAM SỐ HÓA TRONG TỌA ĐỘ PHẲNG
THAM SỐ HÓA TRONG TỌA ĐỘ PHẲNGTHAM SỐ HÓA TRONG TỌA ĐỘ PHẲNG
THAM SỐ HÓA TRONG TỌA ĐỘ PHẲNG
 
Chuyên đề về đường tròn
Chuyên đề về đường trònChuyên đề về đường tròn
Chuyên đề về đường tròn
 
các bài toán hình học lớp 9 có lời giải
các bài toán hình học lớp 9 có lời giảicác bài toán hình học lớp 9 có lời giải
các bài toán hình học lớp 9 có lời giải
 
CHUYÊN ĐỀ QUAN HỆ VUÔNG GÓC - CHƯƠNG 3 HÌNH HỌC 11 (THẦYHOÀNG THÁI VIỆT)
CHUYÊN ĐỀ QUAN HỆ VUÔNG GÓC - CHƯƠNG 3 HÌNH HỌC 11 (THẦYHOÀNG THÁI VIỆT)CHUYÊN ĐỀ QUAN HỆ VUÔNG GÓC - CHƯƠNG 3 HÌNH HỌC 11 (THẦYHOÀNG THÁI VIỆT)
CHUYÊN ĐỀ QUAN HỆ VUÔNG GÓC - CHƯƠNG 3 HÌNH HỌC 11 (THẦYHOÀNG THÁI VIỆT)
 

More from Bui Loi

[Tapchiolympic] Vận dụng tính đơn điệu của hàm số để giải phương trình vô tỷ ...
[Tapchiolympic] Vận dụng tính đơn điệu của hàm số để giải phương trình vô tỷ ...[Tapchiolympic] Vận dụng tính đơn điệu của hàm số để giải phương trình vô tỷ ...
[Tapchiolympic] Vận dụng tính đơn điệu của hàm số để giải phương trình vô tỷ ...
Bui Loi
 
Linear Algebra_ Theory_Jim Hefferon
Linear Algebra_ Theory_Jim HefferonLinear Algebra_ Theory_Jim Hefferon
Linear Algebra_ Theory_Jim Hefferon
Bui Loi
 
Bai Tap Hinh Hoc Vi Phan (CoLoiGiai)
Bai Tap Hinh Hoc Vi Phan (CoLoiGiai)Bai Tap Hinh Hoc Vi Phan (CoLoiGiai)
Bai Tap Hinh Hoc Vi Phan (CoLoiGiai)
Bui Loi
 
Tài liệu bồi dưỡng học sinh giỏi - Chuyên đề 3 - Bài toán liên quan đồ thị
Tài liệu bồi dưỡng học sinh giỏi - Chuyên đề 3 - Bài toán liên quan đồ thị Tài liệu bồi dưỡng học sinh giỏi - Chuyên đề 3 - Bài toán liên quan đồ thị
Tài liệu bồi dưỡng học sinh giỏi - Chuyên đề 3 - Bài toán liên quan đồ thị
Bui Loi
 
[CPO - Tạp chí và tư liệu toán học] Thực tế Nguyên Hàm - Tích Phân
[CPO - Tạp chí và tư liệu toán học] Thực tế Nguyên Hàm - Tích Phân[CPO - Tạp chí và tư liệu toán học] Thực tế Nguyên Hàm - Tích Phân
[CPO - Tạp chí và tư liệu toán học] Thực tế Nguyên Hàm - Tích Phân
Bui Loi
 
Computational Commutative Algebra - Kreuzer and Robbiano
Computational Commutative Algebra - Kreuzer and RobbianoComputational Commutative Algebra - Kreuzer and Robbiano
Computational Commutative Algebra - Kreuzer and Robbiano
Bui Loi
 
Mortad-Mohammed-Hichem-Introductory-topology-exercises-and-solutions-World-Sc...
Mortad-Mohammed-Hichem-Introductory-topology-exercises-and-solutions-World-Sc...Mortad-Mohammed-Hichem-Introductory-topology-exercises-and-solutions-World-Sc...
Mortad-Mohammed-Hichem-Introductory-topology-exercises-and-solutions-World-Sc...
Bui Loi
 
formation_latex
formation_latexformation_latex
formation_latex
Bui Loi
 
Latex 2014 Alexander Borbon
Latex 2014 Alexander BorbonLatex 2014 Alexander Borbon
Latex 2014 Alexander Borbon
Bui Loi
 
Tài lieu tập huấn STEM câp THPT
Tài lieu tập huấn STEM câp THPTTài lieu tập huấn STEM câp THPT
Tài lieu tập huấn STEM câp THPT
Bui Loi
 
(Graduate Texts in Mathematics) Jurgen Herzog, Takayuki Hibi, Hidefumi Ohsugi...
(Graduate Texts in Mathematics) Jurgen Herzog, Takayuki Hibi, Hidefumi Ohsugi...(Graduate Texts in Mathematics) Jurgen Herzog, Takayuki Hibi, Hidefumi Ohsugi...
(Graduate Texts in Mathematics) Jurgen Herzog, Takayuki Hibi, Hidefumi Ohsugi...
Bui Loi
 
dai-so-dong-dieu_nguyen-viet-dong
dai-so-dong-dieu_nguyen-viet-dongdai-so-dong-dieu_nguyen-viet-dong
dai-so-dong-dieu_nguyen-viet-dong
Bui Loi
 
Cơ sở giải tích - Độ đo
Cơ sở giải tích - Độ đo Cơ sở giải tích - Độ đo
Cơ sở giải tích - Độ đo
Bui Loi
 
Giaitichcoso(PGS.TS.NguyenBichHuy).pdf
Giaitichcoso(PGS.TS.NguyenBichHuy).pdfGiaitichcoso(PGS.TS.NguyenBichHuy).pdf
Giaitichcoso(PGS.TS.NguyenBichHuy).pdf
Bui Loi
 
Toán Rời Rạc.pdf
Toán Rời Rạc.pdfToán Rời Rạc.pdf
Toán Rời Rạc.pdf
Bui Loi
 
21 bài tập - TỈ SỐ THỂ TÍCH - có lời giải chi tiết.doc
21 bài tập - TỈ SỐ THỂ TÍCH - có lời giải chi tiết.doc21 bài tập - TỈ SỐ THỂ TÍCH - có lời giải chi tiết.doc
21 bài tập - TỈ SỐ THỂ TÍCH - có lời giải chi tiết.doc
Bui Loi
 
công thức giải nhanh toán 12
 công thức giải nhanh toán 12 công thức giải nhanh toán 12
công thức giải nhanh toán 12
Bui Loi
 
The tich khoi da dien VDVDC
The tich khoi da dien   VDVDCThe tich khoi da dien   VDVDC
The tich khoi da dien VDVDC
Bui Loi
 
Trắc nghiệm nâng cao MATH12
Trắc nghiệm nâng cao MATH12 Trắc nghiệm nâng cao MATH12
Trắc nghiệm nâng cao MATH12
Bui Loi
 
Do do tich-phan-thai_thuan_quang mearsure and intergral
Do do tich-phan-thai_thuan_quang mearsure and intergralDo do tich-phan-thai_thuan_quang mearsure and intergral
Do do tich-phan-thai_thuan_quang mearsure and intergral
Bui Loi
 

More from Bui Loi (20)

[Tapchiolympic] Vận dụng tính đơn điệu của hàm số để giải phương trình vô tỷ ...
[Tapchiolympic] Vận dụng tính đơn điệu của hàm số để giải phương trình vô tỷ ...[Tapchiolympic] Vận dụng tính đơn điệu của hàm số để giải phương trình vô tỷ ...
[Tapchiolympic] Vận dụng tính đơn điệu của hàm số để giải phương trình vô tỷ ...
 
Linear Algebra_ Theory_Jim Hefferon
Linear Algebra_ Theory_Jim HefferonLinear Algebra_ Theory_Jim Hefferon
Linear Algebra_ Theory_Jim Hefferon
 
Bai Tap Hinh Hoc Vi Phan (CoLoiGiai)
Bai Tap Hinh Hoc Vi Phan (CoLoiGiai)Bai Tap Hinh Hoc Vi Phan (CoLoiGiai)
Bai Tap Hinh Hoc Vi Phan (CoLoiGiai)
 
Tài liệu bồi dưỡng học sinh giỏi - Chuyên đề 3 - Bài toán liên quan đồ thị
Tài liệu bồi dưỡng học sinh giỏi - Chuyên đề 3 - Bài toán liên quan đồ thị Tài liệu bồi dưỡng học sinh giỏi - Chuyên đề 3 - Bài toán liên quan đồ thị
Tài liệu bồi dưỡng học sinh giỏi - Chuyên đề 3 - Bài toán liên quan đồ thị
 
[CPO - Tạp chí và tư liệu toán học] Thực tế Nguyên Hàm - Tích Phân
[CPO - Tạp chí và tư liệu toán học] Thực tế Nguyên Hàm - Tích Phân[CPO - Tạp chí và tư liệu toán học] Thực tế Nguyên Hàm - Tích Phân
[CPO - Tạp chí và tư liệu toán học] Thực tế Nguyên Hàm - Tích Phân
 
Computational Commutative Algebra - Kreuzer and Robbiano
Computational Commutative Algebra - Kreuzer and RobbianoComputational Commutative Algebra - Kreuzer and Robbiano
Computational Commutative Algebra - Kreuzer and Robbiano
 
Mortad-Mohammed-Hichem-Introductory-topology-exercises-and-solutions-World-Sc...
Mortad-Mohammed-Hichem-Introductory-topology-exercises-and-solutions-World-Sc...Mortad-Mohammed-Hichem-Introductory-topology-exercises-and-solutions-World-Sc...
Mortad-Mohammed-Hichem-Introductory-topology-exercises-and-solutions-World-Sc...
 
formation_latex
formation_latexformation_latex
formation_latex
 
Latex 2014 Alexander Borbon
Latex 2014 Alexander BorbonLatex 2014 Alexander Borbon
Latex 2014 Alexander Borbon
 
Tài lieu tập huấn STEM câp THPT
Tài lieu tập huấn STEM câp THPTTài lieu tập huấn STEM câp THPT
Tài lieu tập huấn STEM câp THPT
 
(Graduate Texts in Mathematics) Jurgen Herzog, Takayuki Hibi, Hidefumi Ohsugi...
(Graduate Texts in Mathematics) Jurgen Herzog, Takayuki Hibi, Hidefumi Ohsugi...(Graduate Texts in Mathematics) Jurgen Herzog, Takayuki Hibi, Hidefumi Ohsugi...
(Graduate Texts in Mathematics) Jurgen Herzog, Takayuki Hibi, Hidefumi Ohsugi...
 
dai-so-dong-dieu_nguyen-viet-dong
dai-so-dong-dieu_nguyen-viet-dongdai-so-dong-dieu_nguyen-viet-dong
dai-so-dong-dieu_nguyen-viet-dong
 
Cơ sở giải tích - Độ đo
Cơ sở giải tích - Độ đo Cơ sở giải tích - Độ đo
Cơ sở giải tích - Độ đo
 
Giaitichcoso(PGS.TS.NguyenBichHuy).pdf
Giaitichcoso(PGS.TS.NguyenBichHuy).pdfGiaitichcoso(PGS.TS.NguyenBichHuy).pdf
Giaitichcoso(PGS.TS.NguyenBichHuy).pdf
 
Toán Rời Rạc.pdf
Toán Rời Rạc.pdfToán Rời Rạc.pdf
Toán Rời Rạc.pdf
 
21 bài tập - TỈ SỐ THỂ TÍCH - có lời giải chi tiết.doc
21 bài tập - TỈ SỐ THỂ TÍCH - có lời giải chi tiết.doc21 bài tập - TỈ SỐ THỂ TÍCH - có lời giải chi tiết.doc
21 bài tập - TỈ SỐ THỂ TÍCH - có lời giải chi tiết.doc
 
công thức giải nhanh toán 12
 công thức giải nhanh toán 12 công thức giải nhanh toán 12
công thức giải nhanh toán 12
 
The tich khoi da dien VDVDC
The tich khoi da dien   VDVDCThe tich khoi da dien   VDVDC
The tich khoi da dien VDVDC
 
Trắc nghiệm nâng cao MATH12
Trắc nghiệm nâng cao MATH12 Trắc nghiệm nâng cao MATH12
Trắc nghiệm nâng cao MATH12
 
Do do tich-phan-thai_thuan_quang mearsure and intergral
Do do tich-phan-thai_thuan_quang mearsure and intergralDo do tich-phan-thai_thuan_quang mearsure and intergral
Do do tich-phan-thai_thuan_quang mearsure and intergral
 

Recently uploaded

AV6 - PIE CHART WRITING skill in english
AV6 - PIE CHART WRITING skill in englishAV6 - PIE CHART WRITING skill in english
AV6 - PIE CHART WRITING skill in english
Qucbo964093
 
Khoá luận tốt nghiệp ngành Truyền thông đa phương tiện Xây dựng kế hoạch truy...
Khoá luận tốt nghiệp ngành Truyền thông đa phương tiện Xây dựng kế hoạch truy...Khoá luận tốt nghiệp ngành Truyền thông đa phương tiện Xây dựng kế hoạch truy...
Khoá luận tốt nghiệp ngành Truyền thông đa phương tiện Xây dựng kế hoạch truy...
https://www.facebook.com/garmentspace
 
BAI TAP ON HE LOP 2 LEN 3 MON TIENG VIET.pdf
BAI TAP ON HE LOP 2 LEN 3 MON TIENG VIET.pdfBAI TAP ON HE LOP 2 LEN 3 MON TIENG VIET.pdf
BAI TAP ON HE LOP 2 LEN 3 MON TIENG VIET.pdf
phamthuhoai20102005
 
Chương III (Nội dung vẽ sơ đồ tư duy chương 3)
Chương III (Nội dung vẽ sơ đồ tư duy chương 3)Chương III (Nội dung vẽ sơ đồ tư duy chương 3)
Chương III (Nội dung vẽ sơ đồ tư duy chương 3)
duykhoacao
 
GIÁO TRÌNH 2-TÀI LIỆU SỬA CHỮA BOARD MONO TỦ LẠNH MÁY GIẶT ĐIỀU HÒA.pdf
GIÁO TRÌNH 2-TÀI LIỆU SỬA CHỮA BOARD MONO TỦ LẠNH MÁY GIẶT ĐIỀU HÒA.pdfGIÁO TRÌNH 2-TÀI LIỆU SỬA CHỮA BOARD MONO TỦ LẠNH MÁY GIẶT ĐIỀU HÒA.pdf
GIÁO TRÌNH 2-TÀI LIỆU SỬA CHỮA BOARD MONO TỦ LẠNH MÁY GIẶT ĐIỀU HÒA.pdf
Điện Lạnh Bách Khoa Hà Nội
 
98 BÀI LUYỆN NGHE TUYỂN SINH VÀO LỚP 10 TIẾNG ANH DẠNG TRẮC NGHIỆM 4 CÂU TRẢ ...
98 BÀI LUYỆN NGHE TUYỂN SINH VÀO LỚP 10 TIẾNG ANH DẠNG TRẮC NGHIỆM 4 CÂU TRẢ ...98 BÀI LUYỆN NGHE TUYỂN SINH VÀO LỚP 10 TIẾNG ANH DẠNG TRẮC NGHIỆM 4 CÂU TRẢ ...
98 BÀI LUYỆN NGHE TUYỂN SINH VÀO LỚP 10 TIẾNG ANH DẠNG TRẮC NGHIỆM 4 CÂU TRẢ ...
Nguyen Thanh Tu Collection
 
Ảnh hưởng của nhân sinh quan Phật giáo đến đời sống tinh thần Việt Nam hiện nay
Ảnh hưởng của nhân sinh quan Phật giáo đến đời sống tinh thần Việt Nam hiện nayẢnh hưởng của nhân sinh quan Phật giáo đến đời sống tinh thần Việt Nam hiện nay
Ảnh hưởng của nhân sinh quan Phật giáo đến đời sống tinh thần Việt Nam hiện nay
chinhkt50
 
CHUYÊN ĐỀ BỒI DƯỠNG HỌC SINH GIỎI KHOA HỌC TỰ NHIÊN 9 CHƯƠNG TRÌNH MỚI - PHẦN...
CHUYÊN ĐỀ BỒI DƯỠNG HỌC SINH GIỎI KHOA HỌC TỰ NHIÊN 9 CHƯƠNG TRÌNH MỚI - PHẦN...CHUYÊN ĐỀ BỒI DƯỠNG HỌC SINH GIỎI KHOA HỌC TỰ NHIÊN 9 CHƯƠNG TRÌNH MỚI - PHẦN...
CHUYÊN ĐỀ BỒI DƯỠNG HỌC SINH GIỎI KHOA HỌC TỰ NHIÊN 9 CHƯƠNG TRÌNH MỚI - PHẦN...
Nguyen Thanh Tu Collection
 
30 - ĐỀ THI HSG - HÓA HỌC 9 - NĂM HỌC 2021 - 2022.pdf
30 - ĐỀ THI HSG - HÓA HỌC 9 - NĂM HỌC 2021 - 2022.pdf30 - ĐỀ THI HSG - HÓA HỌC 9 - NĂM HỌC 2021 - 2022.pdf
30 - ĐỀ THI HSG - HÓA HỌC 9 - NĂM HỌC 2021 - 2022.pdf
ngocnguyensp1
 
Dẫn luận ngôn ngữ - Tu vung ngu nghia.pptx
Dẫn luận ngôn ngữ - Tu vung ngu nghia.pptxDẫn luận ngôn ngữ - Tu vung ngu nghia.pptx
Dẫn luận ngôn ngữ - Tu vung ngu nghia.pptx
nvlinhchi1612
 

Recently uploaded (10)

AV6 - PIE CHART WRITING skill in english
AV6 - PIE CHART WRITING skill in englishAV6 - PIE CHART WRITING skill in english
AV6 - PIE CHART WRITING skill in english
 
Khoá luận tốt nghiệp ngành Truyền thông đa phương tiện Xây dựng kế hoạch truy...
Khoá luận tốt nghiệp ngành Truyền thông đa phương tiện Xây dựng kế hoạch truy...Khoá luận tốt nghiệp ngành Truyền thông đa phương tiện Xây dựng kế hoạch truy...
Khoá luận tốt nghiệp ngành Truyền thông đa phương tiện Xây dựng kế hoạch truy...
 
BAI TAP ON HE LOP 2 LEN 3 MON TIENG VIET.pdf
BAI TAP ON HE LOP 2 LEN 3 MON TIENG VIET.pdfBAI TAP ON HE LOP 2 LEN 3 MON TIENG VIET.pdf
BAI TAP ON HE LOP 2 LEN 3 MON TIENG VIET.pdf
 
Chương III (Nội dung vẽ sơ đồ tư duy chương 3)
Chương III (Nội dung vẽ sơ đồ tư duy chương 3)Chương III (Nội dung vẽ sơ đồ tư duy chương 3)
Chương III (Nội dung vẽ sơ đồ tư duy chương 3)
 
GIÁO TRÌNH 2-TÀI LIỆU SỬA CHỮA BOARD MONO TỦ LẠNH MÁY GIẶT ĐIỀU HÒA.pdf
GIÁO TRÌNH 2-TÀI LIỆU SỬA CHỮA BOARD MONO TỦ LẠNH MÁY GIẶT ĐIỀU HÒA.pdfGIÁO TRÌNH 2-TÀI LIỆU SỬA CHỮA BOARD MONO TỦ LẠNH MÁY GIẶT ĐIỀU HÒA.pdf
GIÁO TRÌNH 2-TÀI LIỆU SỬA CHỮA BOARD MONO TỦ LẠNH MÁY GIẶT ĐIỀU HÒA.pdf
 
98 BÀI LUYỆN NGHE TUYỂN SINH VÀO LỚP 10 TIẾNG ANH DẠNG TRẮC NGHIỆM 4 CÂU TRẢ ...
98 BÀI LUYỆN NGHE TUYỂN SINH VÀO LỚP 10 TIẾNG ANH DẠNG TRẮC NGHIỆM 4 CÂU TRẢ ...98 BÀI LUYỆN NGHE TUYỂN SINH VÀO LỚP 10 TIẾNG ANH DẠNG TRẮC NGHIỆM 4 CÂU TRẢ ...
98 BÀI LUYỆN NGHE TUYỂN SINH VÀO LỚP 10 TIẾNG ANH DẠNG TRẮC NGHIỆM 4 CÂU TRẢ ...
 
Ảnh hưởng của nhân sinh quan Phật giáo đến đời sống tinh thần Việt Nam hiện nay
Ảnh hưởng của nhân sinh quan Phật giáo đến đời sống tinh thần Việt Nam hiện nayẢnh hưởng của nhân sinh quan Phật giáo đến đời sống tinh thần Việt Nam hiện nay
Ảnh hưởng của nhân sinh quan Phật giáo đến đời sống tinh thần Việt Nam hiện nay
 
CHUYÊN ĐỀ BỒI DƯỠNG HỌC SINH GIỎI KHOA HỌC TỰ NHIÊN 9 CHƯƠNG TRÌNH MỚI - PHẦN...
CHUYÊN ĐỀ BỒI DƯỠNG HỌC SINH GIỎI KHOA HỌC TỰ NHIÊN 9 CHƯƠNG TRÌNH MỚI - PHẦN...CHUYÊN ĐỀ BỒI DƯỠNG HỌC SINH GIỎI KHOA HỌC TỰ NHIÊN 9 CHƯƠNG TRÌNH MỚI - PHẦN...
CHUYÊN ĐỀ BỒI DƯỠNG HỌC SINH GIỎI KHOA HỌC TỰ NHIÊN 9 CHƯƠNG TRÌNH MỚI - PHẦN...
 
30 - ĐỀ THI HSG - HÓA HỌC 9 - NĂM HỌC 2021 - 2022.pdf
30 - ĐỀ THI HSG - HÓA HỌC 9 - NĂM HỌC 2021 - 2022.pdf30 - ĐỀ THI HSG - HÓA HỌC 9 - NĂM HỌC 2021 - 2022.pdf
30 - ĐỀ THI HSG - HÓA HỌC 9 - NĂM HỌC 2021 - 2022.pdf
 
Dẫn luận ngôn ngữ - Tu vung ngu nghia.pptx
Dẫn luận ngôn ngữ - Tu vung ngu nghia.pptxDẫn luận ngôn ngữ - Tu vung ngu nghia.pptx
Dẫn luận ngôn ngữ - Tu vung ngu nghia.pptx
 

Ứng dụng phương tích và trục đẳng phương vào bài toán hình học phẳng

  • 1. Trang 1 BỘ GIÁO DỤC VÀ ĐÀO TẠO VIỆN NGHIÊN CỨU CAO CẤP VỀ TOÁN BÀI BÁO CHỦ ĐỀ HÌNH HỌC PHẲNG ĐỀ TÀI ỨNG DỤNG CỦA PHƯƠNG TÍCH, TRỤC ĐẲNG PHƯƠNG VÀO BÀI TOÁN HÌNH HỌC PHẲNG Văn Phú Quốc GV. Trường THPT chuyên Nguyễn Bỉnh Khiêm, tỉnh Quảng Nam Phương tích và trục đẳng phương là một chủ đề quen thuộc và hấp dẫn trong hình học phẳng. Các kết quả của nó vô cùng đơn giản, tự nhiên nhưng lại ảnh hưởng sâu sắc đến các nội dung quan trọng như: tính độ dài, góc, chứng minh đẳng thức-bất đẳng thức hình học, tập hợp điểm, điểm-đường cố định, quan hệ vuông góc, quan hệ đồng quy-thẳng hàng…Tìm được mối liên hệ giữa phương tích và trục đẳng phương với các nội dung trên sẽ giúp người làm toán hướng đến những lời giải hay, đẹp, gọn gàng và ấn tượng. A. CƠ SỞ LÝ THUYẾT I. Phương tích của một điểm đối với đường tròn (Power of a point) 1. Định lý 1. Cho đường tròn   ; O R và điểm M cố định, đặt OM d  . Một đường thẳng thay đổi cắt đường tròn tại hai điểm A và B. Khi đó: 2 2 2 2 . MA MB MO R d R     . Chứng minh Gọi C là điểm đối xứng của A qua tâm O Vì  0 90 ABC  (góc nội tiếp chắn nửa đường tròn) nên B là hình chiếu của C trên AM . Bây giờ ta có    . . . MA MB MA MB MC MA MO OC MO OA                     2 2 MO OA MO OA MO OA                  2 2 2 2 MO OA d R     2. Định nghĩa. Đại lượng không đổi 2 2 . MA MB d R   được gọi là phương tích của điểm M đối với đường tròn   O , ký hiệu   / M O P . Ta có công thức:   2 2 / . M O MA MB d R    P 3. Các tính chất 3.1. Tính chất 1  Nếu , A B cố định và . AB AM const  thì M cố định.  Điểm M nằm ngoài     / ; 0 M O O R   P . B O M A C
  • 2. Trang 2  Điểm M nằm trong     / ; 0 M O O R   P .  Điểm M nằm trên biên     / ; 0 M O O R   P . 3.2. Tính chất 2 Cho đường tròn   ; O R và một điểm M nằm ngoài đường tròn   O . Qua M kẻ cát tuyến MAB và tiếp tuyến MT với   O . Khi đó 2 2 2 . MA MB MT OM R      . 3.2. Tính chất 3 Cho , AB CD là hai đường thẳng phân biệt sao cho AB CD M   ( , , , A B M C D   ). Khi đó nếu . . MA MB MC MD        thì bốn điểm , , , A B C D cùng nằm trên một đường tròn. Chứng minh Giả sử đường tròn ngoại tiếp ABC  cắt CD tại D . Khi đó: . . MA MB MC MD MD MD D D                 . Vậy bốn điểm , , , A B C D cùng nằm trên một đường tròn. 3.3. Tính chất 4 Cho , AB MT là hai đường thẳng phân biệt sao cho AB MT M   (với , , M A B T   ). Khí đó nếu 2 . MA MB MT    thì đường tròn ngoại tiếp ABT  tiếp xúc với MT tại T . 4. Một số ví dụ cơ bản 3.1. Ví dụ 1. Cho ABC  nội tiếp đường tròn   ; O R và G là trọng tâm ABC  . Chứng minh rằng:     2 2 2 / 1 9 G O AB BC CA     P . Lời giải Vì G là trọng tâm ABC  nên 3OG OA OB OC           . Suy ra   2 2 2 2 9 2 . . . OG OA OB OC OAOB OB OC OC OA                    2 3 2 . . . R OAOB OB OC OC OA               (1) Ta có   2 2 2 2 2 2 2 2 2 . 2 OAOB OA OB OA OB OA OB AB R AB                      . Tương tự 2 2 2 . 2 OB OC R BC       ; 2 2 2 . 2 OC OA R CA      Suy ra     2 2 2 2 2 . . . 6 OAOB OB OC OC OA R AB BC CA                 (2) Thay (2) vào (1) ta được:     2 2 2 2 2 2 2 2 2 2 1 9 9 9 OG R AB BC CA OG R AB BC CA           . Do đó     2 2 2 / 1 9 G O AB BC CA     P . [Phương tích này được gọi là phương tích trọng tâm]
  • 3. Trang 3 3.2. Ví dụ 2. Cho ABC  nội tiếp đường tròn   ; O R và H là trực tâm ABC  . Chứng minh rằng:   2 / 8 cos cos cos H O R A B C   P . Lời giải Đối với bài toán này, ta quan tâm đến trường hợp ABC  nhọn, còn những trường hợp vuông hoặc tù ta chứng minh tương tự. Gọi , I A lần lượt là giao điểm của AH với   , BC O . Áp dụng định lý sin trong HAB  ta có       0 0 sin 90 sin 180 sin sin AH AB AH AB A C ABH AHB      .cos 2 cos cos sin sin AH AB AB HA A R A A C C      . Tương tự ta cũng có 2 cos HB R B  ; 2 cos HC R C  . Vì    BHA C BA A     nên BHA  cân tại B. Suy ra I là trung điểm A H  . Khi đó  2 2. .cos 4 cos cos      HA IH HB BHA R B C . Vì ABC  nhọn nên   2 / . . 8 cos cos cos H O HA HA HA HA R A B C        P . [Phương tích này được gọi là phương tích trực tâm] 3.3. Ví dụ 3. Cho đường tròn   ; O R và ba điểm thẳng hàng , , A B C . Chứng minh đẳng thức:       / / / . . . . . 0 A O B O C O BC CA AB BC CA AB     P P P . Lời giải Ta có:       / / / . . . . . 0 A O B O C O BC CA AB BC CA AB     P P P       2 2 2 2 2 2 . . . . . OA R BC OB R CA OC R AB BC CA AB          2 2 2 2 . . . . . OA BC OB CA OC AB BC CA AB R BC CA AB        2 2 2 . . . . . OA BC OB CA OC AB BC CA AB     Ta sẽ chứng minh hệ thức Stewart 2 2 2 . . . . . 0 OA BC OB CA OC AB BC CA AB     Trường hợp 1. Điểm O nằm trên đường thẳng chứa ba điểm , , A B C . 2 2 2 . . . . .    OA BC OB CA OC AB BC CA AB           2 2 2 OA OC OB OB OA OC OC OB OA OC OB OA OC OB OA           2 2 2 2 2 2 2 2 . . . . . . . . OA OC OA OB OB OA OB OC OC OB OC OA OA OC OA OB         2 2 2 2 . . . . . . . . 0 OB OA OB OC OC OB OC OA OAOB OC OAOB OC        . I C' B' A' H O B C A
  • 4. Trang 4 Trường hợp 2. Điểm O không nằm trên đường thẳng chứa ba điểm , , A B C . Gọi H là hình chiếu của điểm O lên đường thẳng chứa ba điểm , , A B C . 2 2 2 . . . . .    OA BC OB CA OC AB BC CA AB       2 2 2 2 2 2 . . . . OH HA BC OH HB CA OH HC AB BC CA AB            2 2 2 2 . . . . . OH BC CA AB AH BC BH CA CH AB BC CA AB        0  . Vậy       / / / . . . . . 0 A O B O C O BC CA AB BC CA AB     P P P . 4. Phương tích trong hệ tọa độ Descartes Trong mặt phẳng với hệ tọa độ Oxy , cho điểm   0 0 ; M x y và đường tròn   2 2 : 2 2 0 C x y ax by c      . Đặt   2 2 , 2 2 f x y x y ax by c      . Khi đó     2 2 0 0 0 0 0 0 / ; 2 2 M C f x y x y ax by c       P . II. Trục đẳng phương của hai đường tròn (Radical axis) và tâm đẳng phương (radical center) 1. Định lý 2. Cho hai đường tròn không đồng tâm     1 1 2 2 ; , ; O R O R . Tập hợp điểm M có phương tích đối với hai đường tròn bằng nhau là một đường thẳng. Đường thẳng này được gọi là trục đẳng phương của hai đường tròn   1 O và   2 O . Chứng minh Giả sử điểm M có cùng phương tích đối với hai đường tròn   1 O và   2 O . Gọi H là hình chiếu của điểm M lên đường thẳng 1 2 O O và I là trung điểm đoạn thẳng 1 2 O O . Khi đó     2 2 2 2 2 2 1 2 1 2 MH HO MH HO R R      2 2 2 2 1 2 1 2 HO HO R R     H O A C B I O1 O2 H M
  • 5. Trang 5    2 2 1 2 1 2 1 2 HO HO HO HO R R      2 2 2 2 1 2 2 1 1 2 1 2 .2 2 R R O O HI R R IH O O       . Vì H là điểm cố định nên tập hợp các điểm M có cùng phương tích với hai đường tròn là đường thẳng qua H và vuông góc với 1 2 O O . 2. Các tính chất về trục đẳng phương Xét hai đường tròn   1 O và   2 O . Từ kết quả ở định lý 2 ta suy ra một số tính chất sau: 2.1. Tính chất 1. Trục đẳng phương của hai đường tròn vuông góc với đường thẳng nối tâm. 2.2. Tính chất 2. Nếu hai đường tròn cắt nhau tại A và B thì AB chính là trục đẳng phương của chúng. 2.3. Tính chất 3. Nếu     1 2 / / M O M O  P P thì đường thẳng qua M vuông góc với 1 2 O O là trục đẳng phương của hai đường tròn. 2.4. Tính chất 4. Nếu     1 2 / / M O M O  P P và     1 2 / / N O N O  P P thì đường thẳng MN chính là trục đẳng phương của hai đường tròn. 2.5. Tính chất 5. Nếu     1 2 / / M O M O  P P ;     1 2 / / N O N O  P P và     1 2 / / P O P O  P P thì ba điểm , , M N P thẳng hàng. 2.6. Tính chất 6. Nếu   1 O và   2 O tiếp xúc nhau tại A thì đường thẳng qua A và vuông góc với 1 2 O O chính là trục đẳng phương của hai đường tròn. 3. Cách dựng trục đẳng phương của hai đường tròn không đồng tâm Trong mặt phẳng, cho hai đường tròn không đồng tâm   1 O và   2 O . Xét các khả năng có thể xảy ra: 3.1. Khả năng 1.       1 2 , O O A B   . Khi đó đường thẳng AB chính là trục đẳng phương của hai đường tròn. 3.2. Khả năng 2.   1 O và   2 O tiếp xúc nhau tại A. Khi đó tiếp tuyến chung tại A chính là trục đẳng phương của hai đường tròn. 3.3. Khả năng 3.     1 2 O O    . Ta dựng theo các bước dưới đây:  Bước 1: Dựng đường tròn   3 O sao cho       3 1 , O O A B   và       3 2 , O O C D   .  Bước 2: AB CD M   .  Bước 3: Dựng đường thẳng qua M và vuông góc với đường thẳng 1 2 O O chính là trục đẳng phương của   1 O và   2 O . 4. Trục đẳng phương trong hệ tọa độ Descartes Trong mặt phẳng với hệ tọa độ Oxy , cho hai đường tròn không đồng tâm:   2 2 1 1 1 1 : 2 2 0 C x y a x b y c      và   2 2 2 2 2 2 : 2 2 0 C x y a x b y c     
  • 6. Trang 6 (với 2 2 0 i i i a b c    , 1,2 i  ). Giả sử điểm   0 0 ; M x y . Ta có     1 2 2 2 2 2 0 0 2 0 2 0 2 0 0 2 0 2 0 2 / / 2 2 2 2 M C M C x y a x b y c x y a x b y c            P P     1 2 0 1 2 0 1 2 2 2 0 a a x b b y c c        . Vậy trục đẳng phương của hai đường tròn   1 C và   2 C có phương trình:     1 2 1 2 1 2 2 2 0 a a x b b y c c       . 5. Định lý 3. Cho ba đường tròn     1 2 , O O và   3 O . Khi đó ba trục đẳng phương của các cặp đường tròn hoặc trùng nhau hoặc song song hoặc cùng đi qua một điểm. Điểm đó là tâm đẳng phương của ba đường tròn. Chứng minh Gọi ij  là trục đẳng phương của hai đường tròn   i O và   j O . Ta xem xét các trường hợp sau đây: Trường hợp 1. Giả sử có một cặp đường thẳng song song. Không giảm tính tổng quát, ta có thể giả sử 12  // 23  . Ta có 12 1 2 23 2 3 , O O O O     . Điều này dẫn đến 1 2 3 , , O O O thẳng hàng. Lại có 13 1 3 O O   nên 13  // 23  // 12  . Trường hợp 2. Giả sử có một cặp đường thẳng cắt nhau. Không giảm tính tổng quát, ta có thể giả sử 12 23 M     . 13 23 12 M O1 O2 O3 Bây giờ ta có             1 2 1 3 2 3 1 / / / 3 / / / M O M O M O M O M O M O M            P P P P P P . Bắt đầu từ đây, ta suy ra: - Nếu có hai đường thẳng trùng nhau thì đó cũng là trục đẳng phương của cặp đường tròn còn lại. - Nếu hai trục đẳng phương chỉ cắt nhau tại một điểm thì điểm đó cũng thuộc trục đẳng phương còn lại.
  • 7. Trang 7 6. Các tính chất về tâm đẳng phương 6.1. Tính chất 1. Nếu ba đường tròn đôi một cắt nhau thì các dây cung cùng đi qua một điểm. 6.2. Tính chất 2. Nếu ba trục đẳng phương song song hoặc trùng nhau thì tâm của ba đường tròn thẳng hàng. 6.3. Tính chất 3. Nếu ba đường tròn cùng đi qua một điểm và có các tâm thẳng hàng thì các trục đẳng phương trùng nhau. 7. Một số ví dụ cơ bản 7.1. Ví dụ 1. Cho ABC  , một đường thẳng song song với BC cắt , AB AC tương ứng tại , D E . Xác định trục đẳng phương của các đường tròn đường kính , BE CD. Lời giải Gọi     1 2 , C C lần lượt là các đường tròn đường kính , BE CD. Gọi , , AA BB CC    là các đường cao của ABC  và H là trực tâm. Dễ thấy các tứ giác HA CB  , HA BC   nội tiếp, cho nên . . . . . . AB AC AH AA AB AC AC AB AC AB AH AA                (1) Vì DE // BC nên AC AB AE AD  (2) Từ (1) và (2) suy ra . '. AB AE AC AD       1 2 / / A C A C   P P Suy ra A nằm trên trục đẳng phương của hai đường tròn   1 C và   2 C . Hơn nữa, ta thấy AA vuông góc với đường nối tâm của hai đường tròn   1 C và   2 C Vậy AA là trục đẳng phương của   1 C và   2 C . 7.2. Ví dụ 2. Cho ABC  có đường cao BD và CE cắt nhau tại H . Gọi M là trung điểm BC , N là giao điểm của DE và BC . Chứng minh rằng: NH AM  . Lời giải Gọi , O I lần lượt là trung điểm của các đoạn thẳng , AH MH . Ta có:     DEH DAH DBC FEH        2. 2. FED FEH DBC DMC     Suy ra tứ giác EDMF nội tiếp. Điều đó dẫn đến . . NE ND NF NM  tức điểm N nằm trên trục đẳng phương của đường tròn   ; O OH và đường tròn   ; I IH . E D A' B' C' H B A C I O N M D E F H B A C
  • 8. Trang 8 Mặt khác,     ; ; H O OH I IH   . Do đó NH chính là trục đẳng phương của hai đường tròn   O và   I . Suy ra NH OI  , mà OI // AM nên NH AM  . Vậy ta hoàn tất việc chứng minh. 7.3. Ví dụ 3. Cho đường tròn   O đường kính AB và CD . Tiếp tuyến tại B của   O cắt AC tại E , DE cắt   O tại điểm thứ hai F . Chứng minh rằng , , AF BC OE đồng quy. Lời giải Ký hiệu     1 2 , C C tương ứng là đường tròn ngoại tiếp của các tam giác , AEF BCE . Ta có , AF BC tương ứng là trục đẳng phương của   O và   1 C ,   O và   2 C . Mặt khác    OAF FDB FEA   ;   OBC CEB  . Suy ra , OA OB lần lượt là tiếp tuyến của     1 2 , C C . Kết hợp với 2 2 OA OB  ta suy ra OE là trục đẳng phương của   1 C và   2 C . Suy ra , , AF BC OE đồng quy tại tâm đẳng phương của ba đường tròn. [tư tưởng của định lý 3]. B. MỘT SỐ ỨNG DỤNG I. Ứng dụng 1: Chứng minh một số định lý về hình học phẳng 1.1. [Định lý Brianchon]. Cho ABCDEF là một lục giác ngoại tiếp đường tròn   O . Khi đó , , AD BE CF đồng quy. Chứng minh Gọi , , , , , G H I J K L tương ứng là tiếp điểm của , , , , , AB BC CD DE EF FA với   O . Trên các tia , , , , , KF HB GB JD ID LF lần lượt lấy các điểm , , , , , P S Q R N M sao cho KP SH GQ JR IN LM      . Dựng đường tròn   1 O tiếp xúc với , EF CB tại , P S ; đường tròn   2 O tiếp xúc với , AF CD tại , M N và đường tròn   3 O tiếp xúc với , AB ED tại , Q R. Ta có FP PK FK LM LF FM      CS SH HC IN IC CN      F E O B A D C N M R Q S P C D E F A B I J G H K L O1 O3 O2
  • 9. Trang 9 Suy ra FC là trục đẳng phương của các đường tròn   1 O và   2 O . Chứng minh tương tự, ta cũng thu được: AD là trục đẳng phương của các đường tròn   2 O và   3 O ; BE là trục đẳng phương của   3 O và   1 O . Áp dụng định lý 3 [về tâm đẳng phương] ta đi đến , , AD BE CF đồng quy. 1.2. [Hệ thức Euler]. Cho ABC  nội tiếp đường tròn   ; O R và ngoại tiếp đường tròn   ; I R . Chứng minh rằng 2 2 2 OI R Rr   . Chứng minh Gọi E là tiếp điểm của AB và   ; I r ; gọi   ' ; A AI O R   . Ta có  sin sin 2 IE r IA A IAE   (do AA là phân giác  BAC ). Mặt khác,       0 180 2 2 A B BIA AIE BIE IBA         A IB   cân tại A IA BA      . Áp dụng sin cho BAA  ta có: 2 sin 2 sin 2 2 A A BA R IA R      . Vì I là điểm nằm bên trong   ; O R nên   2 2 2 2 / . .2 sin 2 2 2 2 sin 2 I O r A IA IA R Rr OI R Rr OI R Rr A               P . 1.3. [Định lý Fuss]. Cho tứ giác ABCD nội tiếp đường tròn   ; O R và ngoại tiếp đường tròn   ; I r . Đặt 0 OI d   . Khi đó     2 2 2 1 1 1 r R d R d     . Chứng minh Kéo dài , BI DI cắt   O tại , M N . Ta có:     MNC IBC NMC IDC                0 1 90 2 MNC NMC IBC IDC ABC ADC        O  là trung điểm đoạn thẳng MN . Sử dụng công thức đường trung tuyến trong IMN  có E A' I O C A B I F E O C A D B M N
  • 10. Trang 10 2 2 2 2 2 2 2 2 2 4 2 IM IN MN IM IN d OI R        Suy ra 2 2 2 2 2 IM IN R d    . Do đó             2 2 2 2 2 2 2 2 2 2 / 2 1 1 I O R d IM IN R d R d R d         P 2 2 2 2 2 2 2 2 1 1 . . IM IN IM IB IN ID IB ID         2 2 2 2 2 2 2 sin sin sin sin 1 2 2 2 ABC ADC IBE IDF IE IF r      . II. Ứng dụng 2: Chứng minh quan hệ vuông góc 2.1. [Mathematical Olympiad Summer Program (MOSP) 1995]. Cho ABC  có đường cao BD và CE cắt nhau tại H . M là trung điểm của BC , N là giao điểm của DE và BC . Chứng minh rằng NH AM  . Lời giải Gọi , O I tương ứng là trung điểm của , AH MH . Ta có     DEH DAH DBC FEH        2. 2. FED FEH DBC DMC     tức giác EDMF nội tiếp . . NE ND NF NM N    nằm trên trục đẳng phương của đường tròn đường kính MH và đường tròn đường kính AH . Mặt khác,     H O I NH    là trục đẳng phương của   O và   I . Suy ra NH OI  . Mà OI // AM (do OI là đường trung bình AHM  ) nên NH AM  . 2.2. [Iran 1996]. Cho hai điểm , D E tương ứng nằm nằm trên các cạnh , AB AC của ABC  sao cho DE // BC . Gọi P là điểm bất kì nằm bên trong ABC  ; các đường thẳng , PB PC lần lượt cắt DE tại F và G . Gọi 1 2 , O O lần lượt là tâm đường tròn ngoại tiếp các tam giác , PDG PFE . Chứng minh 1 2 AP O O  . O I N M E D F H A B C
  • 11. Trang 11 Lời giải Gọi M , N lần lượt là giao điểm thứ hai của AB với   1 O , của AC với   2 O . Ta có    PMD PGD PCB    tứ giác BPCM nội tiếp. Tương tự ta cũng chứng minh được tứ giác BPCN nội tiếp. Điều đó dẫn đến tứ giác BMNC nội tiếp. Mà DE // BC cho nên ta thu được tứ giác MDEN nội tiếp. Áp dụng định lý 3 [về tâm đẳng phương] cho các đường tròn     , DGP PEF và   DENM ta có DM EN A   nằm trên trục đẳng phương của   1 O và   2 O . P nằm trên trục đẳng phương AP  là trục đẳng phương của   1 O và   2 O . Do đó 1 2 AP OO  (điều phải chứng minh). 2.3. [IMO 1985]. Cho ABC  . Đường tròn tâm O đi qua các điểm , A C và lại cắt các đoạn , AB BC theo thứ tự tại hai điểm phân biệt , K N . Giả sử đường tròn ngoại tiếp của các tam giác ABC và KBN cắt nhau tại B và M . Chứng minh  0 90 OMB  . Lời giải Gọi D AC KN   . Ta có          0 0 180 180 KMA BMA KMB ACB KNB          KNB ACB KDA    . tứ giác AKMD nội tiếp     0 0 180 180 AMD AKD ACB AMB       Do đó , , B M D thẳng hàng. Gọi R là bán kính của đường tròn   O . Ta có   2 2 2 2 . . * . . BM BD BK BA BO R DM DB DC DA DO R              2 2 2 . 2 BM DM BD BO DO R      2 2 2 2 2 BD BO DO R     . M N O2 O1 P E A B C D F G M D K N A B C O
  • 12. Trang 12 Từ (*) ta suy ra: 2 2 2 2 2 2 2 2 BO R DO R BM DM BD BD                     2 2 2 2 2 2 2 BO DO R BO DO BD      2 2 0 90 BO DO BMO     . 2.4. [Belarus 2000]. Gọi P là giao điểm của đường chéo AC và BD của tứ giác lồi ABCD trong đó AB AC BD   . Gọi O và I lần lượt là tâm đường tròn ngoại tiếp và nội tiếp của ABP  . Nếu minh rằng nếu O không trùng với I thì OI CD  . Lời giải Đầu tiên ta chứng minh một luận đề hết sức hữu ích về hai đường vuông góc. Cho hai đường XY và UV , đặt , X Y   lần lượt là chân đường vuông góc của X và Y tương ứng, nối đường thẳng UV . Sử dụng trực tiếp khoảng cách, XY UV  nếu và chỉ nếu UX X V UY Y V        . Vì UX X V UV UY Y V         , phép tính trên đạt được nếu và chỉ nếu 2 2 2 UX X V UY Y V        hay 2 2 2 2 UX XV UY YV    . Do đó nó thỏa mãn đẳng thức 2 2 2 2 DO CO DI CI    . Đặt , AB AC BD p PC a     và PD b  . Thế thì AP p a   và BP p b   . Gọi R là bán kính đường tròn ngoại tiếp ABP  . Sử dụng định lý về phương tích của một điểm, ta có 2 2 . pb DP DB DO R    . Tương tự, 2 2 pa CO R   . Do đó   2 2 DO CO p b a    . Vì ABD  là cân với BA BD  và I nằm trên đường phân giác của  ABD ID IA   . Ngoài ra IB IC  . Gọi T là điểm tiếp xúc của đường tròn nội tiếp ABC  với cạnh AB . Thế thì 2 2 AB AP BP p b a AT       và 2 p a b BT    bởi vì 2 2 2 2 , IT AB AI BI AT BT     . Đặt các tham số lại với nhau chúng ta thấy rằng      2 2 2 2 2 2 2 2 . DI CI AI BI AT BT AT BT AT BT p b a DO CO             Suy ra OI CD  . III. Ứng dụng 3: Chứng minh quan hệ đồng quy, thẳng hàng 3.1. [Vietnam TST 2009]. Cho ABC  nhọn nội tiếp đường tròn   O . Các điểm 1 1 1 , , A B C lần lượt là chân đường vuông góc của , , A B C xuống các cạnh đối diện. Các điểm 2 2 2 , , A B C đối xứng với 1 1 1 , , A B C qua trung điểm , , BC CA AB . Đường tròn ngoại T I O P D C B A
  • 13. Trang 13 tiếp các tam giác 2 2 2 2 2 2 , , AB C BC A CA B cắt   O lần thứ hai tại 3 3 3 , , A B C . Chứng minh các đường thẳng 1 3 1 3 1 3 , , A A B B C C đồng quy. Lời giải Gọi   ; I R là đường tròn ngoại tiếp 2 2 AB C  , M là trung điểm AB và 1 3 G AM A A   . Ta có 1 1 1 2 1 2 . . , AC AB AB AC AC BC AB CB         2 2 . . BC BA CB CA   2 2 2 2 BI R CI R     BI CI   IO BC   Mà 3 IO AA  cho nên BC // 3 AA . Dễ dàng có 1 3 1 2 A M AA  . Theo định lý Thales ta có 3 1 2 AA AG MG A M   . Suy ra G là trọng tâm ABC  . Chứng minh tương tự, ta cũng suy ra được 1 3 1 3 , B B C C cũng đi qua G . Vậy 1 3 1 3 1 3 , , A A B B C C đồng quy tại trọng tâm G của ABC  . 3.2. [IMO Shortlist 2013, Thailand]. Cho ABC  nhọn với H là trực tâm. Gọi W là điểm trên cạnh BC . Gọi , M N tương ứng là chân đường cao kẻ từ đỉnh B và C . Gọi 1  là đường tròn ngoại tiếp BWN  và X là điểm trên 1  đầu mút của đường kính, đối diện với W . Tương tự, 2  là đường tròn ngoại tiếp CWM  và Y là điểm trên 2  đầu mút của đường kính, đối diện với W . Chứng minh rằng , , X Y H thẳng hàng. Lời giải Gọi L là chân đường cao kẻ từ A và Z là giao điểm thứ hai khác W của hai đường tròn 1 2 ,   . Ta sẽ chứng minh , , X Y Z và H cùng nằm trên một đường thẳng. Từ   0 90 BNC BMC   ta suy ra , , B C N và M cùng nằm trên một đường tròn, gọi đường tròn này là 3  . Để ý rằng , , WZ BN CM tương ứng là trục đẳng phương của các cặp đường tròn 1  và 2  ; 1  và 3  ; 2  và 3  . Z N L M H A B C W X Y G A3 I C2 B2 M A1 B1 C1 O C B A A2
  • 14. Trang 14 Thế thì A BN CM   là tâm đẳng phương của ba đường tròn và do đó WZ qua A. Vì , WX WY tương ứng là đường kính của 1  và 2  nên ta có   0 90 WZX WZY   , vì vậy , X Y nằm trên đường thẳng qua Z , vuông góc với WZ . Vì tứ giác BLHN có hai góc đối diện vuông nên nội tiếp đường tròn. Từ phương tích của điểm A đối với các đường tròn 1  và   BLHN ta được: . . . AL AH AB AN AW AZ   . Nếu H nằm trên đường thẳng AW thì H Z  lập tức. Mặt khác, vì AZ AL AH AW  nên các tam giác , AHZ AWL đồng dạng. Vì thế,   0 90 HZA WLA   . Vậy H cũng nằm trên đường thẳng XYZ . 3.3.[Iran NMO 2001]. Cho ABC  nội tiếp đường tròn   O . Gọi     , a I I lần lượt là đường tròn nội tiếp, bàng tiếp góc A. Giả sử a I I cắt BC và   O lần lượt tại , A M    M A   . Gọi N là trung điểm cung  MBA và , a NI NI cắt   O lần lượt tại , S T . Chứng minh rằng , , S T A thẳng hàng. Lời giải           1 1 2 2 NTS NA AS NM AS NIM        a a I TS I IS    tứ giác a I TIS nội tiếp đường tròn và ký hiệu đường tròn này là   1  . Mặt khác,   0 90 a a IBI ICI   nên a IBI C nội tiếp đường tròn và ký hiệu đường tròn này là   2  . Nhận thấy , , a I I BC TS tương ứng là trục đẳng phương của các cặp đường tròn   1  và   2  ,   O và   2  ,   O và   1  . Do đó , , a I I BC TS đồng quy tại A. Vậy ba điểm , , S T A thẳng hàng. 3.4. [IMO 1995]. Trên đường thẳng d lấy bốn điểm , , , A B C D (theo thứ tự đó). Đường tròn đường kính AC và BD cắt nhau tại , X Y . Đường thẳng XY cắt BC tại Z . Lấy P là một điểm trên XY khác Z . Đường thẳng CP cắt đường tròn đường kính AC tại điểm thứ hai là M và BP cắt đường tròn đường kính BD tại điểm thứ hai N . Chứng minh rằng , , AM DN XY đồng quy. T N A' M Ia I O B C A S
  • 15. Trang 15 Lời giải  Nếu P X  thì , M X N X   . Khi đó , , AM DN XY đồng quy.  Nếu P Y  thì chứng minh tương tự trường hợp trên.  Nếu , X P Y P     . Gọi J AM XY   , J DN XY    . Tứ giác JMCZ nội tiếp nên . . PJ PZ PM PC  . Tương tự . . PJ PZ PN PB   . Do P nằm trên trục đẳng phương của đường tròn đường kính AC và đường tròn đường kính BD nên . . PM PC PN PB  . . PJ PZ PJ PZ J J       . Vậy , , AM DN XY đồng quy. 3.5. [Luyện thi Quốc gia, Quốc tế của TP.Hồ Chí Minh]. Cho hai điểm , A B phân biệt nằm trên đường tròn   O và C nằm ngoài   O . Gọi CS và CT là các tiếp tuyến của C với   O với , S T là các tiếp điểm, M là trung điểm của cung nhỏ AB . Các đường thẳng , MS MT cắt AB lần lượt tại , E F . Đường thẳng đi qua , E F vuông góc với AB cắt OS , OT lần lượt tại , X Y . Một đường thẳng bất kì qua C cắt   O tại , P Q ( P nằm giữa C và Q ). Gọi R là giao của MP với AB , Z là tâm đường tròn ngoại tiếp tam giác   PQR . Chứng minh rằng , , X Y Z thẳng hàng. Lời giải Không mất tính tổng quát, ta giả sử E nằm ngoài đoạn AB . Ta thấy rằng EX //OM , mà OSM  cân tại O nên XES  cân tại X . Xét đường tròn ( , ) X XE , dễ thấy rằng đường tròn này tiếp xúc với   O tại S và SC là tiếp tuyến chung trong của hai đường tròn này. Ta có:   MAB MPA  nên hai MAR  và MPA  đồng dạng. Tương tự, MAE  và MSA  cũng đồng dạng. Từ đó suy ra , MA MR MA ME MP MA MS MA   hay . . MP MR MS ME  . Suy ra M nằm trên trục đẳng phương của đường tròn ( ) X và ( ) PQR . J M N Z Y X A D B C P Z R P Y X F E M T S O C A B Q
  • 16. Trang 16 Ta cũng có . CS CP CQ  2 nên C cũng nằm trên trục đẳng phương của hai đường tròn ( ) X và ( ) PQR . Từ đó suy ra CM là trục đẳng phương của ( ) X và ( ) PQR hay CM XZ  . Hoàn toàn tương tự, ta cũng có CM YZ  . Vậy , , X Y Z thẳng hàng. IV. Ứng dụng 4: Chứng minh một tập hợp điểm cùng nằm trên một đường tròn 4.1. [APMO 2009]. Trong cùng một mặt phẳng, cho ba đường tròn 1 2 3 , ,    không đè lên nhau và tiếp nhau ngoài nhau. Với mỗi điểm P trong mặt phẳng đó và nằm ngoài cả ba vòng tròn, ta dựng sáu điểm 1 1 2 2 3 3 , , , , , A B A B A B theo cách sau đây: Với mọi 1,2,3 i  các điểm , i i A B phân biệt và cùng nằm trên i  sao cho các đường thẳng , i i PA PB đều tiếp xúc với i  . Ta gọi điểm P là điểm ngoại lệ nếu từ cách dựng trên, ba đường thẳng 1 1 2 2 3 3 , , A B A B A B đồng quy. Chứng minh rằng mọi điểm ngoại lệ nếu có cùng nằm trên một đường tròn. Lời giải Gọi i O và i r lần lượt là tâm và bán kính của i  với 1,2,3 i  . Giả sử P là điểm ngoại lệ và ba đường thẳng 1 1 2 2 3 3 , , A B A B A B đồng quy tại Q . Dựng đường tròn đường kính PQ . Gọi  là đường tròn tâm O bán kính r . Ta sẽ chứng minh mọi điểm ngoại lệ đều nằm trên  . Giả sử 1 1 1 1 PO A B X   . Vì 1 1 1 PO A B  nên 1 X nằm trên  . Do 1 PA là một tiếp tuyến của 1  nên 1 1 PAO  vuông và đồng dạng với 1 1 1 A X O  . Suy ra 1 1 1 1 1 1 1 O X O A O A O P  hay 2 2 1 1 1 1 1 1 . O X O P O A r   . Mặt khác, 1 1 1 . O X O P cũng là phương tích của điểm 1 O đối với  nên    2 2 2 1 1 1 1 1 1 1 . r O X O P O O r O O r OO r       và do đó    2 2 2 1 1 1 1 1 1 r OO r OO r OO r      . Như vậy 2 r chính là phương tích của điểm O đối với 1  . Lý luận tương tự 2 r cũng là phương tích của điểm O đối với đường tròn 2  và 3  . Suy ra O phải nằm trên tâm đẳng phương của ba đường tròn đã cho. Do r là căn bậc hai phương tích của điểm O đối với ba đường tròn đã cho nên r không phụ thuộc vào vị trí điểm P. Điều này có nghĩa là mọi điểm ngoại lệ đều nằm trên  . r X1 O P O1 A1 B1 Q
  • 17. Trang 17 Nhận xét. Trong trường hợp tâm đẳng phương của ba đường tròn nằm tại vô cùng (khi đó ba trục đẳng phương song song với nhau), sẽ không có điểm ngoại lệ nào nằm trong mặt phẳng, điều này đã giải thích từ “nếu có” ở đề bài. 4.2. [IMO 2008]. Cho ABC  nhọn với trực tâm H . Gọi 1 2 3 , , M M M lần lượt là trung điểm , , BC CA AB . Đường tròn tâm 1 M bán kính 1 M H cắt BC tại 1 2 , A A ; đường tròn tâm 2 M bán kính 2 M H cắt AC tại 1 2 , B B ; đường tròn tâm 3 M bán kính 3 M H cắt AB tại 1 2 , C C . Chứng minh rằng sáu điểm 1 2 1 2 1 2 , , , , , A A B B C C cùng thuộc một đường tròn. Lời giải Do 1 2 M M // AB ( 1 2 M M đường trung bình) và AB HC  ( H trực tâm) nên 1 2 M M HC  . 1 2 1 2 . . CA CA CB CB   Bốn điểm 1 2 1 2 , , , A A B B thuộc đường tròn và gọi đường tròn này là   1  . Chứng minh tương tự ta thu được: - Bốn điểm 1 2 1 2 , , , A A B B thuộc đường tròn   2  - Bốn điểm 1 2 1 2 , , , C C B B thuộc đường tròn   3  Nếu sáu điểm 1 2 1 2 1 2 , , , , , A A B B C C không cùng thuộc một đường tròn thì các trục đẳng phương của ba đường tròn       1 2 3 , ,    phải đồng quy tại một điểm nhưng chúng lại cắt nhau tại , , A B C nên đây là điều vô lý. Vậy mệnh đề phản chứng là sai, ta đi đến kết luận của bài toán. 4.3.[IMO Shortlist 2006, Ukraine]. Cho hình thang ABCD với hai cạnh song song AB CD  . Hai điểm K và L nằm trên các đoạn thẳng AB và CD theo thứ tự thỏa mãn AK DL KB LC  . Giả sử rằng có các điểm P và Q nằm trên đoạn KL thỏa mãn   APB BCD  và   CQD ABC  . Chứng minh rằng các điểm , , , P Q B C cùng thuộc một đường tròn. Lời giải Từ giả thiết, ta có , , AK DL AD BC KL BK CL   đồng quy tại E . Dựng đường tròn   1 O qua hai điểm , C D và tiếp xúc với BC ; dựng đường tròn   2 O qua hai điểm , A B và tiếp xúc với BC . Khi đó    DQC ABC DCE   nên   1 Q O  . Chứng minh tương tự ta cũng được   2 P O  . Gọi F là giao điểm thứ hai của EQ với   1 O . B2 B1 A2 A1 C2 C1 H M2 M3 M1 A B C
  • 18. Trang 18 Ta có 2 . EF EQ EC  (1) Mặt khác,   1 2 2 1 O CD O BA AO B DO C    1 1 2 2 , , O C DC EC k E O O O B AB EB      thẳng hàng và 1 1 2 2 EO k EO kEO EO        . Suy ra phép vị tự       1 2 , : E k V O O  . Mà , , E F P thẳng hàng,     1 2 , O F P O   dẫn đến EF EC EF kEP k EP EB         (2) Từ (1) và (2) suy ra . . EP EQ EC EB  . Vậy bốn điểm , , , P Q B C cùng thuộc một đường tròn. 4.4. [International Zhautykov Olympiad 2008]. Trong mặt phẳng cho hai đường tròn   1 O và   2 O . Gọi 1 2 A A là tiếp tuyến chung của hai đường tròn       1 1 2 2 , A O A O   , K là trung điểm 1 2 A A . Từ K lần lượt kẻ hai tiếp tuyến 1 2 , KB KB đến     1 2 , O O . Gọi 1 1 2 2 1 2 , L A B A B P KL O O     . Chứng minh rằng 1 2 , , , B B P L cùng nằm trên một đường tròn. Lời giải Do 1 2 1 2 KA KA KB KB    nên tứ giác 1 1 2 2 A B B A nội tiếp đường tròn 1 1 2 2 . . LB LA LB LA   KL  là trục đẳng phương của hai đường tròn   1 O và   2 O 1 2 KL O O   . Để ý rằng ba điểm 1 1 , , A B P nhìn đoạn 1 O K dưới một góc 0 90 nên nội tiếp. Tương tự, ta cũng chứng minh được tứ giác 2 2 A B PK nội tiếp. Áp dụng định lý Miquel ta suy ra tứ giác 1 2 B PB L nội tiếp. Vậy 1 2 , , , B B P L cùng nằm trên một đường tròn. F Q P L E A B D C K O2 O1 P L K A1 O1 O2 A2 B1 B2
  • 19. Trang 19 V. Ứng dụng 5: Chứng minh bài toán điểm-đường cố định 5.1. [VMO 2014]. Cho ABC  nhọn nội tiếp đường tròn   O , trong đó , B C cố định và A thay đổi trên   O . Trên các tia AB và AC lần lượt lấy các điểm M và N sao cho MA MC  và NA NB  . Các đường tròn ngoại tiếp các tam giác AMN và ABC cắt nhau tại P   P A   . Đường thẳng MN cắt BC tại Q . a) Chứng minh rằng ba điểm , , A P Q thẳng hàng. b) Gọi D là trung điểm của BC . Các đường tròn có tâm là , M N và cùng đi qua A cắt nhau tại K   K A   . Đường thẳng qua A vuông góc với AK cắt BC tại E . Đường tròn ngoại tiếp ADE  cắt   O tại F   F A   . Chứng minh rằng AF đi qua một điểm cố định. Lời giải a) Không mất tính tổng quát, ta giả sử AB AC  như hình vẽ, các trường hợp còn loại hoàn toàn tương tự. Khi đó, M nằm ngoài đoạn AB và N nằm trong đoạn AC . Vì NA NB  nên   NBA NAB  ; lại vì MA MC  nên   MCA MAC  . Suy ra   NBA MCA  hay tứ giác BMCN nội tiếp và ta được: . . QM QN QB QC  . Từ đây suy ra Q có cùng phương tích đến hai đường tròn   O và   AMN nên nó nằm trên trục đẳng phương AP của hai đường tròn này. Do đó , , A P Q thẳng hàng. b) Ta thấy rằng đường tròn   ODC tiếp xúc với   O tại C nên trục đẳng phương của hai đường tròn này là tiếp tuyến d của   O tại C . Ta cần chứng minh   O ADE  . Thật vậy, ta có , O M cùng nằm trên trung trực của AC nên OM AC  . Tương tự, ON AB  cho nên O là trực tâm AMN  . Suy ra AO MN  . Xét hai đường tròn     , , , M MA N NA ta thấy AK MN  . Suy ra , , A O K thẳng hàng nên  0 90 OAE  . Mặt khác,  0 90 ODE  nên tứ giác AODE nội tiếp hay   O ADE  . Do đó, trục đẳng phương của   ADE và   ODC chính là OD . Hơn nữa, trục đẳng phương của   O và   ADE là AF . d I F E Q K P O D M A B C N
  • 20. Trang 20 Nhận thấy ba đường tròn       , , O ADE ODC đôi một có các trục đẳng phương là , , OD d AF nên theo định lý 3 [về tâm đẳng phương] ba đường này đồng quy tại một điểm. Vậy AF đi qua giao điểm I OD d   và I là một điểm cố định. 5.2. [Chọn đội tuyển phổ thông năng khiếu (PTNK) 2008]. Cho tam giác ABC có đỉnh A cố định và , B C thay đổi trên đường thẳng d cố định sao cho nếu gọi A là hình chiếu của A lên d thì . A B A C   âm và không đổi. Gọi M là hình chiếu của A lên AB . Gọi N là hình chiếu của A lên AC . Gọi K là giao điểm của các tiếp tuyến của đường tròn ngoại tiếp A MN   tại M và N . Chứng minh rằng K thuộc đường thẳng cố định. Lời giải Ta có 2 . . AM AB AA AN AC    tứ giác BMNC nội tiếp   AMN ACB   . Lại có   ADB ACB  nên   AMN ADB  . Điều này dẫn đến tứ giác MPDB nội tiếp. Do đó 2 . . AP AD AM AB AA   . Mà , , A A D  cố định nên P cố định. Gọi H là hình chiếu của K trên AA. Khi đó 2 2 1 . . 4 OP OH OI OK ON AA    . Mà , , , A P A O  cố định nên H cố định. Vậy K thuộc đường thẳng qua H và vuông góc với AA. 5.3. [VMO 2003]. Cho đường tròn   1 1 ; O R tiếp xúc ngoài với đường tròn   2 2 ; O R tại M với 2 1 R R  . Xét điểm A di động trên đường tròn   2 2 ; O R sao cho 1 2 , , A O O không thẳng hàng. Từ A kẻ tiếp tuyến , AB AC đến   1 O . Các đường thẳng , MB MC cắt   2 O tại , E F . Gọi D là giao điểm của EF với tiếp tuyến tại A của đường tròn   2 O . Chứng minh rằng D di động trên một đường thẳng cố định. Lời giải Qua M kẻ tiếp tuyến chung của   1 O và   2 O . Ta có     MCA CMy FMD FAM    FAM FCA   (g.g) 2 2 2 1 1 . FA FM FC FO R     (1) Tương tự, 2 2 2 1 1 EA EO R   (2) H P D I K O N M A' A B C y A M O2 O1 B C E F D
  • 21. Trang 21 Ký hiệu   ;0 A là đường tròn tâm A, bán kính 0 R  . Từ (1) và (2) ta suy ra EF là trục đẳng phương của   ;0 A với   1 O . Mà D nằm trên EF nên     1 2 2 2 2 1 1 / / D O D O DA DO R P P     . Vậy D nằm trên trục đẳng phương của hai đường tròn cố định   1 O và   2 O . 5.4.[Đề đề nghị kỳ thi Olympic duyên hải và đồng bằng Bắc Bộ năm 2014]. Cho ABC  vuông ở A, đường cao AH . Gọi , E F theo thứ tự là hình chiếu của H trên , AB AC . Chứng minh rằng: Khi , A H không thay đổi còn , B C thay đổi thì đường tròn ngoại tiếp tứ giác BCFE luôn đi qua 2 điểm cố định. Lời giải Theo giả thiết, trong các tam giác vuông AHB và AHC ta có: 2 . AE AB AH  và 2 . AF AC AH  . Do đó: . . AE AB AF AC  . Suy ra tứ giác BECF nội tiếp. Gọi , P Q là giao điểm của AH với đường tròn   BEFC . Do tứ giác BPCQ nội tiếp nên: . . HP HQ HB HC  . Trong tam giác vuông ABC có: 2 . HB HC AH  . Do đó 2 . . HP HQ HB HC AH   . Mặt khác: 2 . . AP AQ AE AB AH   . Suy ra 2 ( )( ) AH HP AH HQ AH    2 2 ( ) . . AH AH HQ HP HP HQ AH AH HQ HP         Do đó: 2 . HP HQ AH HP HQ AH         . Giải hệ này ta được: 5 1 2 5 1 2 HP AH HQ AH             , P Q cố định. Vậy đường tròn   BEFC luôn đi qua 2 điểm cố định , P Q. P Q E F H A B C
  • 22. Trang 22 VI. Ứng dụng 6: Tính toán các đại lượng hình học 6.1. [USA 1998]. Cho hai đường tròn đồng tâm O ký hiệu là   1 C và   2 C (  2 C nằm trong   1 C ). Từ một điểm A nằm trên   1 C kẻ tiếp tuyến AB đến   2 C . AB cắt   2 C lần thứ hai tại C . D là trung điểm của AB . Một đường thẳng qua A cắt   2 O tại , E F sao cho đường trung trực của DF và EC giao nhau tại điểm M nằm trên AC . Tính tỉ số AM MC . Lời giải Rõ ràng B là trung điểm AC . Ta có   2 2 / 1 . .2 . 2 A C AE AF AB AB AB AD AC P     . Suy ra tứ giác DCFE nội tiếp đường tròn. Do đó M là tâm đường tròn ngoại tiếp tứ giác DCFE . Mà M nằm trên AC nên 1 2 MD MC DC   . Điều đó giúp ta tính được 5 3 4 3 5 4 AM AB AM MC MC AB                  . 6.2. [Russian MO 2008]. Cho ABC  nội tiếp đường tròn   ; O R , ngoại tiếp đường tròn   ; I r . Đường tròn   I tiếp xác với , AB AC lần lượt tại , X Y . Gọi K là điểm chính giữa cung AB không chứa C . Giả sử XY chia đôi đoạn AK . Tính  BAC ? Lời giải Gọi S XY AK   . Ta có   KAB KCB  ;    0 180 SXA AXY BIC AXS CIB      . AS AX AX CB AS CB CI CI     1 . 2 AX BC AK CI   hay . 2 . KI CI AX BC  . Đặt , , , , 2 ABC a b c BC a CA b AB c S S p         Ta có       2 2 2 2 2 . 4 4 abc S Rr p a a p a a bc p p a b c bc a S p             M E D C A O B F I O B A C X Y K S
  • 23. Trang 23 Mặt khác, theo định lý hàm côsin ta có 2 2 2 2 cos a b c bc A    . Khi đó 2 2 2 2 0 1 2 cos cos 120 2 b c bc b c bc A A A          . Vậy  0 120 BAC  . 6.3. [Russian MO 2007]. Cho hai đường tròn   1 O và   2 O cắt nhau tại , A B . Gọi , PQ RS là hai tiếp tuyến chung của hai đường tròn       1 2 ; , , O P R O Q S   . Giả sử RB// PQ , RB cắt   2 O lần nữa tại W . Tính tỉ số RB BW ? Lời giải Gọi J AB PQ   . Suy ra J là trung điểm của đoạn thẳng PQ . Rõ ràng AB // PR; PQ // RB nên tứ giác PJBR là hình bình hành PJ RB   2 2 2 2 . 4 4 RB RW RS PQ PJ RB      4 RW RB   . Vậy 1 3 RB RW  . VII. Ứng dụng 7: Chứng minh một số mệnh đề về góc của tam giác và tìm quỹ tích điểm. 7.1. [Belarus 2000, problem 1]. Cho tứ giác ABCD có hai đường chéo cắt nhau ở M . Đường phân giác của  ACD cắt tia BA ở K . Chứng minh rằng: nếu . . . MA MC MACD MB MD   thì   BKC CDB  . Lời giải Gọi N CK BD   . Áp dụng định lý về đường phân giác cho MCD  ta có: . CD MC MC DN CD DN MN MN    . Khi đó: . . . . MC DN MB MD MA MC MA MN       . 1 . . DN MD MA MC MA MC MN MN          hay . . MA MC MB MN  . S Q B A O1 O2 P R J W N K M A D C B
  • 24. Trang 24 Vì M nằm trong tứ giác ABCN , theo định lý về phương tích thì , , , A B C N cùng nằm trên đường tròn. Từ đó      KBD ABN ACN NCD KCD     . Suy ra , , , K B C D cùng nằm trên một đường tròn. Do đó ta có   BKC CDB  . 7.2.[Belarus 2000, problem 2]. Cho ABC  và  2 C   . Gọi M là trung điểm của cạnh huyền AB , H là chân đường cao CH và P là điểm nằm trong tam giác sao cho AP AC  . Chứng minh rằng PM là phân giác  .BPH khi và chi  3 A   . Lời giải Điểm P nằm trên đường tròn tâm A bán kính AC . Đường tròn  cắt các đường thẳng , CH MH và PH tương ứng tại , D N và Q . Vì MA MC  ,  0 60 A  khi và chỉ khi ACM  đều. Nghĩa là khi và chỉ khi M N  . Điều đó khẳng định PM là phân giác  BPH khi và chỉ khi M N  . Thật vậy, AH là đường cao thuộc đáy của tam giác cân ACD nên H là trung điểm của CD và CD là một dây cung của đường tròn  . Theo định lý về phương tích của một điểm ta có 2 . . PH HQ CH HD CH   . Vì CH là đường cao thuộc cạnh huyền của ABC  nên 2 . CH AH HB  . Như vậy . . PH HQ AH HB  . Do H AB PQ   nên tứ giác APBQ nội tiếp. Để ý trên đường tròn  rằng:     2 2 QAB QAN QPN HPN    . Thế thì     2 HPB QPB QAB HPN    . Vì N HB PN   (PN là phân giác  HPB ). Do đó PM là phân giác  HPB khi và chỉ khi M N  . 7.3. [APMO 1995]. Cho tứ giác lồi cố định ABCD nội tiếp trong đường tròn mà AB không song song với CD . Tìm quỹ tích các điểm P thỏa mãn điều kiện: ta có thể tìm được hai đường tròn lần lượt nhận AB và CD làm dây cung sao cho chúng tiếp xúc nhau tại P . Q N D M H A C B P
  • 25. Trang 25 Lời giải Giả sử AB CD X   . Từ X kẻ một tiếp tuyến đến đường tròn ngoại tiếp tứ giác ABCD và gọi R là độ dài đoạn tiếp tuyến này. Như vậy quỹ tích cần tìm là đường tròn tâm X , bán kính R. Thật vậy, ta gọi   0 C là đường tròn tâm X bán kính R. Lấy một điểm P bất kì trên   0 C . Xét đường tròn ngoại tiếp tứ giác ABCD ta có: 2 . . R XA XB XC XD   . Suy ra 2 . . XP XA XB XC XD   . Gọi   1 C là đường tròn đi qua ba điểm , , C D P , ta có 2 . XC XD XP  nên XP là tiếp tuyến của đường tròn   1 C . Tương tự gọi   2 C là đường tròn đi qua ba điểm , , A B P thì XP là tiếp tuyến của đường tròn này. Như vậy   1 C và   2 C tiếp xúc nhau tại P. Ta chú ý rằng nếu P là một trong bốn giao điểm của AB hoặc CD với   0 C thì không thể dựng các đường tròn như trên được, trừ khi ta chấp nhận rằng đó là đường tròn suy biến thành đường thẳng. Như vậy ta đã chứng minh được rằng tất cả các điểm nằm trên   0 C đều là các điểm của quỹ tích (hoặc trừ đi bốn điểm nói trên nếu không chấp nhận trường hợp suy biến). Bây giờ đảo lại, ta sẽ chứng minh rằng không có điểm nào của quỹ tích mà không nằm trên   0 C . Thật vậy, với bất kì đường tròn nào đi qua hai điểm , C D, chỉ có hai đường tròn qua , A B và tiếp xúc với nó (điều này rõ ràng nếu ta xét cách mà đường tròn qua , A B thay đổi khi tâm của nó di chuyển trên đường trung trực của AB ), do vậy có nhiều nhất là hai điểm thuộc quỹ tích nằm trên đường tròn đi qua , C D cho trước. Mà đây là hai giao điểm của đường tròn đó với   0 C , vậy không có điểm nào của quỹ tích mà không nằm trên   0 C . X A B C D
  • 26. Trang 26 C. BÀI TẬP ĐỀ NGHỊ 1. Cho đường tròn   O tiếp xúc đường thẳng d tại H . Hai điểm , M N di động trên d sao cho 2 .   HM HN k ( 0  k cho trước ). Từ , M N kẻ tiếp tuyến MA và NB của   O (với ,   A B H ). Chứng minh rằng: a) Đường tròn   OMN luôn đi qua 2 điểm cố định. b) Đường thẳng AB luôn đi qua 1 điểm cố định. 2. Cho AB và AC là các tiếp tuyến của đường tròn   O với , B C thuộc   O . Lấy điểm M bất kì trên AC ( , M A khác phía so với C ). Giả sử (O) cắt đường tròn (ABM) tại điểm thứ hai , P Q là chân đường vuông góc hạ từ C xuống MB . Chứng minh rằng:   2 MPQ AMB  . 3. Cho tam giác ABC có đường tròn tâm I nội tiếp, tiếp xúc các cạnh , , BC CA AB tại , , D E F . AI cắt đường tròn   I tại , M N (M nằm giữa A và N ). DM cắt cạnh EF tại K , NK cắt đường tròn   I tại điểm P khác N . Chứng minh rằng các điểm , , A P D thẳng hàng. 4. Cho tứ giác ABCD nội tiếp đường tròn tâm O bán kính R. Các đường thẳng AB và CD cắt nhau tại P, đường thẳng AD và đường thẳng BC cắt nhau tại Q . Chứng minh rằng:  2 2 cos R POQ OP OQ         . 5. Cho tứ giác ABCD nội tiếp đường tròn   ; O R . Gọi , , P Q M lần lượt là giao điểm của các cặp đường thẳng AB và DC , AD và BC , AC và BD . Chứng minh rằng bán kính đường tròn ngoại tiếp các tam giác OPQ,OMP và OMQ bằng nhau. 6. Cho đường tròn tâm O đường kính AB . Một điểm H thuộc đoạn AB . Đường thẳng qua H cắt đường tròn tại C . Đường tròn đường kính CH cắt AC , BC và   O lần lượt tại , D E và F . a) Chứng minh rằng , AB DE và CF đồng quy. b) Đường tròn   ; C CH cắt   O tại P và Q . Chứng minh rằng , , , P D E Q thẳng hàng. 7. Cho tam giác ABC , D thuộc đoạn BC , E thuộc đoạn AD . Đường tròn ngoại tiếp BDE  cắt AB tại K . Đường tròn ngoại tiếp CDE  cắt AC tại L . Gọi M là giao điểm của DK với BE , N là giao điểm của DL với CE , O là tâm đường tròn ngoại tiếp tam giác EBC . Chứng minh rằng AO MN  . ===HẾT===